Licchavi Lyceum

ll

Licchavi Lyceum

UPSC Prelims 2013 Question Paper [Solved]

You are reading the solution of SET A of UPSC Prelims 2013 Question Paper.

1. What will follow if a Money Bill is substantially amended by the Rajya Sabha?

(a) The Lok Sabha may still proceed with the Bill, accepting or not accepting the recommendations of the Rajya Sabha
(b) The Lok Sabha cannot consider the Bill further
(c) The Lok Sabha may send the Bill to the Rajya Sabha for reconsideration
(d) The President may call a joint sitting for passing the Bill

Ans: (a)

Solution: If the Rajya Sabha makes any recommendations/ amendments to the Money Bill, the Lok Sabha may either accept or reject all or any of the recommendations. If the Rajya Sabha does not return the Bill within 14 days, the Bill is deemed to have been passed by both Houses of Parliament in the form in which it was passed by the Lok Sabha.

2. Which one of the following statements is correct?

(a) In India, the same person cannot be appointed as Governor for two or more States at the same time
(b) The Judges of the High Court of the States in India are appointed by the Governor of the State just as the Judges of the Supreme Court are appointed by the President
(c) No procedure has been laid down in the Constitution of India for the removal of a Governor from his/her post
(d) In the case of a Union Territory having a legislative setup, the Chief Minister is appointed by the Lt. Governor on the basis of majority support

Ans: (c)

Solution: S1: Same person can be appointed as the governor of more tan two states. Hence, S1 is incorrect.

S2: The judge of the High Courts are appointed by the President of India based on the recommendation of the recommendations of the collegium of Supreme Court of India.

S3: Correct Statement, there is no such provision available in the Indian Constitution. As the government at the centre changes, many governors are removed from the post. Hence S3 is correct.

S4: The Chief Minister of the UTs are appointed by the President of India. The oath is administered by LG. Hence, S4 is incorrect.

3. Which one of the following pairs is correctly matched?

Geographical Feature   –   Region
(a) Abyssinian Plateau : Arabia
(b) Atlas Mountains : North-Western Africa
(c) Guiana Highlands : South-Western Africa
(d) Okavango Basin : Patagonia

Ans: (B)

Solution: 

Abyssinian Plateau
Abyssinian Plateau
Atlas Mountain
Atlas Mountain
Guiana Highlands
Guiana Highlands
Okavango Basin
Okavango Basin

4. With reference to the history of Indian rock-cut architecture, consider the following statements:

1. The caves at Badami are the oldest surviving rock-cut caves in India.
2. The Barabar rock-cut caves were originally made for Ajivikas by Emperor Chandragupta Maurya.
3. At Ellora, caves were made for different faiths.

Which of the statements given above is/are correct?

(a) 1 only
(b) 2 and 3 only
(c) 3 only
(d) 1, 2 and 3

Ans: (c)

Solution: 

S1: Badami Cave is located in Karnataka and it is ancient rock-cut architecture site. The caves are a testimony to the rich cultural heritage of India, and are believed to have been built during the 6th and 8th centuries by the Chalukya dynasty. It is not the oldest once further younger than the Barbar Caves of Bihar. S1 is incorrect. 

S2: The Barabar rock-cut caves are a group of ancient rock-cut caves located in the Barabar hills in Bihar, India. They were created in the 3rd century BCE during the Mauryan period. The caves were originally made for the Ajivika religious sect, which was a popular sect during that time.

The caves are considered to be one of the earliest surviving examples of rock-cut architecture in India. The caves are known for their unique architectural style and their polished, smooth surfaces. The caves were excavated out of granite and are believed to have been created by skilled craftsmen using only hand tools.

The caves are believed to have been visited by the great Mauryan emperor Ashoka, who was a patron of the Ajivika sect.

Hence, S2 is incorrect. 

S3: The Ellora Caves are a complex of ancient rock-cut temples and monasteries located in the Aurangabad district of Maharashtra, India. The complex includes 34 caves, which are divided into three groups – Buddhist, Hindu, and Jain.

The Buddhist caves are the oldest, and they consist of caves 1 to 12. The most famous among them is the Cave 10, which is also known as the Visvakarma cave. This cave contains some of the finest Buddhist sculptures and paintings.

The Hindu caves consist of caves 13 to 29, and they are dedicated to various Hindu deities such as Shiva, Vishnu, and Durga. The most famous among them is the Cave 16, which is also known as the Kailash temple. This cave is considered to be the most impressive of all the Ellora caves and is dedicated to Lord Shiva.

The Jain caves consist of caves 30 to 34, and they are dedicated to the Jain Tirthankaras. The most famous among them is the Cave 32, which is also known as the Indra Sabha. This cave is famous for its elaborate carvings and sculptures.

The Ellora Caves are significant for their artistic and architectural value. They are considered to be one of the greatest examples of Indian rock-cut architecture. The caves are also significant for their historical importance, as they reflect the religious harmony and cultural diversity of ancient India. In 1983, the Ellora Caves were designated as a UNESCO World Heritage Site.

The Ellora Caves were built by various dynasties that ruled the region during the 6th to the 10th century CE. The Buddhist caves (Caves 1-12) were built during the reign of the Kalachuri dynasty, which ruled the region between the 6th and 7th centuries CE. The Hindu caves (Caves 13-29) were built during the reign of the Rashtrakuta dynasty, which ruled the region between the 8th and 10th centuries CE. The Jain caves (Caves 30-34) were built during the reign of the Yadava dynasty, which ruled the region in the 10th century CE.

5. Recombinant DNA technology (Genetic Engineering) allows genes to be transferred

1. across different species of plants
2. from animals to plants
3. from microorganisms to higher organisms

Select the correct answer using the codes given below.

(a) 1 only
(b) 2 and 3 only
(c) 1 and 3 only
(d) 1, 2 and 3

Ans: (d)

Solution: Recombinant DNA technology, also known as genetic engineering, allows genes to be transferred not only within the same species but also between different species. This technology has made it possible to create new organisms with desirable traits by transferring genes from one organism to another.

This has numerous applications in fields such as agriculture, medicine, and industry. With recombinant DNA technology, genes can be transferred across different species of plants, from animals to plants, and from microorganisms to higher organisms.

6. The Chinese traveller Yuan Chwang (Hiuen Tsang) who visited India recorded the general conditions and culture of India at that time. In this context, which of the following statements is/are correct?

1. The roads and river-routes were completely immune from robbery.
2. As regards punishment for offences, ordeals by fire, water and poison were the instruments for determining the innocence or guilt of a person.
3. The tradesmen had to pay duties at ferries and barrier stations.

Select the correct answer using the codes given below.

(a) 1 only
(b) 2 and 3 only
(c) 1 and 3 only
(d) 1, 2 and 3

Ans: (b)

Solution: Yuan Chwang (Hiuen Tsang) was a Chinese Buddhist monk and traveller who visited India during the 7th century CE. He recorded his observations about Indian society, culture, and religion in his travelogue, the “Records of the Western World”.

S1: According to him the roads were not safe for travel and robbery was common. Hence S1 is incorrect.

S2: The ordeals by fire, water, and poison were the instruments for determining the innocence or guilt of a person. This is a correct statement.

S3: Yuan Chwang did mention that traders had to pay duties at ferries and barrier stations, which was a common practice during that time.

7. Consider the following

1. Star tortoise
2. Monitor lizard
3. Pygmy hog
4. Spider monkey

Which of the above found in India?

(a) 1, 2 and 3 only
(b) 2 and 3 only
(c) 1 and 4 only
(d) 1, 2, 3 and 4

Ans: (a)

Solution: The star tortoise, the monitor lizard and, Pygmy hog are found in India.

8. Which of the following can be found as pollutants in the drinking water in some parts of India?

1. Arsenic
2. Sorbitol
3. Fluoride
4. Formaldehyde
5. Uranium

Select the correct answer using the codes given below.

(a) 1 and 3 only
(b) 2, 4 and 5 only
(c) 1, 3 and 5 only
(d) 1, 2, 3, 4 and 5

Ans: (c)

Solution: Arsenic, fluoride, and uranium are all known to occur as pollutants in drinking water in some parts of India. Sorbitol and formaldehyde are not typically found as pollutants in drinking water.

9. With reference to Indian History, the Members of the Constituent Assembly from the Provinces were

(a) directly elected by the people of those Provinces
(b) nominated by the Indian National Congress and the Muslim League
(c) elected by the Provincial Legislative Assemblies
(d) selected by the Government for their expertise in constitutional matters

Ans: (c)

Solution: The Members of the Constituent Assembly from the Provinces were elected by the Provincial Legislative Assemblies. Dr. Rajendra Prasad was the first Chairman of the Constituent Assembly of India.

10. Consider the following animals:

1. Sea cow
2. Sea horse
3. Sea lion

Which of the above is/are mammal/mammals?

(a) 1 only
(b) 1 and 3 only
(c) 2 and 3 only
(d) 1, 2 and 3

Ans: (b)

Solution: Sea cows and sea lions are both marine mammals, which means they are warm-blooded vertebrates that breathe air, nurse their young with milk, and have hair or fur. Sea cows are also known as manatees and are herbivorous animals that can be found in shallow waters near the coast or in estuaries. Sea lions, on the other hand, are carnivorous and are typically found in coastal waters or on rocky islands.

Sea horses, on the other hand, are not mammals, but rather are a type of fish. They are characterized by their unique appearance, with a horse-like head, a long snout, and a prehensile tail that can grasp objects.

11. Consider the following statements:

1. An amendment to the Constitution of India can be initiated by an introduction of a bill in the Lok Sabha only.
2. If such an amendment seeks to make changes in the federal character of the Constitution, the amendment also requires to be ratified by the legislature of all the States of India.

Which of the statements given above is/are correct?
(a) 1 only
(b) 2 only
(c) Both 1 and 2
(d) Neither 1 nor 2

Answer: (c) Both 1 and 2

Explanation: Statement 1 is correct as an amendment to the Constitution of India can be initiated by introducing a bill in either House of Parliament (Lok Sabha or Rajya Sabha). However, Statement 2 is also correct as some amendments that affect the federal character of the Constitution require ratification by the legislatures of half of the states in India.

12. Consider the following statements: Attorney General of India can

  1. take part in the proceedings of the Lok Sabha
  2. be a member of a committee of the Lok Sabha
  3. speak in the Lok Sabha
  4. vote in the Lok Sabha

Which of the statements given above is/are correct?
(a) 1 only
(b) 2 and 4
(c) 1, 2 and 3
(d) 1 and 3 only

Answer: (d) 1 and 3 only

Explanation: The Attorney General of India can take part in the proceedings of the Lok Sabha (Statement 1) and can also speak in the Lok Sabha (Statement 3). However, the Attorney General cannot be a member of any parliamentary committee and cannot vote in the Lok Sabha as he/she is not a Member of Parliament.

13. With reference to the usefulness of the by-products of sugar industry, which of the following statements is / are correct?

1. Bagasse can be used as biomass fuel for the generation of energy.
2. Molasses can be used as one of the feed-stocks for the production of synthetic chemical fertilizers.
3. Molasses can be used for the production of ethanol.

Select the correct answer using the codes given below.

(a) 1 only
(b) 2 and 3 only
(c) 1 and 3 only
(d) 1, 2 and 3

Answer: (c) 1 and 3 only

Explanation: Statement 1 is correct as bagasse, a by-product of the sugar industry, can be used as biomass fuel for the generation of energy. Statement 2 is incorrect as molasses are used for the production of alcohol (ethanol), not synthetic chemical fertilizers. Statement 3 is correct as molasses are a primary raw material for the production of ethanol.

14. Variations in the length of daytime and nighttime from season to season are due to

(a) the earth’s rotation on its axis
(b) the earth’s revolution round the sun in an elliptical manner
(c) latitudinal position of the place
(d) revolution of the earth on a tilted axis

Answer: (a) the earth’s rotation on its axis

Explanation: Variations in the length of daytime and nighttime from season to season are primarily due to the Earth’s rotation on its axis. This rotation causes different parts of the Earth to receive varying amounts of sunlight, leading to changes in the duration of daytime and nighttime.

15. The Narmada river flows to the west, while most other large peninsular rivers flow to the east. Why?

  1. It occupies a linear rift valley.
  2. It flows between the Vindhyas and the Satpuras.
  3. The land slopes to the west from Central India.

Select the correct answer using the codes given below.
(a) 1 only
(b) 2 and 3
(c) 1 and 3
(d) None

Answer: (c) 1 and 3

Explanation: The Narmada river flows to the west because it occupies a linear rift valley. It flows between the Vindhyas and the Satpura mountain ranges. The land slopes to the west from Central India, which contributes to the westward flow of the river.

16. On the planet Earth, most of the freshwater exists as ice caps and glaciers. Out of the remaining freshwater, the largest proportion

(a) is found in the atmosphere as moisture and clouds
(b) is found in freshwater lakes and rivers
(c) exists as groundwater
(d) exists as soil moisture

Answer: (c) exists as groundwater

Explanation: On Earth, most of the freshwater is locked in ice caps and glaciers. Out of the remaining freshwater, the largest proportion exists as groundwater, which is water stored beneath the Earth’s surface in porous rocks and aquifers.

17. Consider the following pairs:

  1. Nokrek Biosphere Reserve: Garo Hills
  2. Loktak (Logtak) Lake: Barail Range
  3. Namdapha National Park: Dafla Hills

Which of the above pairs is/are correctly matched?
(a) 1 only
(b) 2 and 3 only
(c) 1, 2 and 3
(d) None

Answer: (a) 1 only

Explanation: The Nokrek Biosphere Reserve is correctly matched with Garo Hills. However, Loktak (Logtak) Lake is located in Manipur and not in Barail Range. Namdapha National Park is situated in Arunachal Pradesh and not in Dafla Hills.

18. Consider the following:

  1. Electromagnetic radiation
  2. Geothermal energy
  3. Gravitational force
  4. Plate movements
  5. Rotation of the earth
  6. Revolution of the earth

Which of the above are responsible for bringing dynamic changes on the surface of the earth?
(a) 1, 2, 3, and 4 only
(b) 1, 3, 5, and 6 only
(c) 2, 4, 5, and 6 only
(d) 1, 2, 3, 4, 5, and 6

Answer: (d) 1, 2, 3, 4, 5, and 6

Explanation: All the mentioned factors are responsible for bringing dynamic changes on the surface of the Earth. Electromagnetic radiation from the Sun influences the weather and climate. Geothermal energy contributes to volcanic activity and tectonic plate movements. Gravitational force affects ocean tides and landforms. Plate movements, rotation, and revolution of the Earth also cause changes in the landscape over time.

19. Which of the following bodies does not/do not find mention in the Constitution?

  1. National Development Council
  2. Planning Commission
  3. Zonal Councils

Select the correct answer using the codes given below.
(a) 1 and 2 only
(b) 2 only
(c) 1 and 3 only
(d) 1, 2 and 3

Answer: (b) 2 only

Explanation: The Planning Commission does not find mention in the Constitution. It was a non-constitutional body that was responsible for formulating five-year plans and overseeing the country’s development until it was replaced by the NITI Aayog in 2015. However, the National Development Council and Zonal Councils find mention in the Constitution. The National Development Council is mentioned in Article 263, and the Zonal Councils are provided for in Chapter VIII (Article 263) of the Constitution.

20. The demand for the Tebhaga Peasant Movement in Bengal was for

(a) the reduction of the share of the landlords from one-half of the crop to one-third
(b) the grant of ownership of land to peasants as they were the actual cultivators of the land
(c) the uprooting of the Zamindari system and the end of serfdom
(d) writing off all peasant debts**

Answer: (a) the reduction of the share of the landlords from one-half of the crop to one-third

Explanation: The Tebhaga Peasant Movement in Bengal, which took place in the late 1940s, demanded the reduction of the share of the landlords from one-half (50%) of the crop to one-third (33.3%). The movement sought to improve the condition of the sharecroppers (tenants) who were facing exploitative practices by the landlords. The demand was for a two-thirds share for the peasants and one-third share for the landlords.

21. The Parliament can make any law for the whole or any part of India for implementing international treaties

(a) with the consent of all the States
(b) with the consent of the majority of States
(c) with the consent of the States concerned
(d) without the consent of any State

Answer: (d) without the consent of any State

Explanation: The Parliament can make any law for the whole or any part of India for implementing international treaties without the consent of any State. This power is derived from the authority of the Central Government to conduct foreign affairs and implement international agreements.

22. In the grasslands, trees do not replace the grasses as a part of an ecological succession because of

(a) insects and fungi
(b) limited sunlight and paucity of nutrients
(c) water limits and fire
(d) None of the above

Answer: (b) limited sunlight and paucity of nutrients

Explanation: In grasslands, trees do not replace grasses as a part of ecological succession mainly due to the limited sunlight and paucity of nutrients. Grasslands are adapted to a regime of frequent fires and grazing, which restricts the growth of trees.

23. Which one of the following is the correct sequence of ecosystems in the order of decreasing productivity?

(a) Oceans, lakes, grasslands, mangroves
(b) Mangroves, oceans, grasslands, lakes
(c) Mangroves, grasslands, lakes, oceans
(d) Oceans, mangroves, lakes, grasslands

Answer: (a) Oceans, lakes, grasslands, mangroves

Explanation: The correct sequence of ecosystems in the order of decreasing productivity is Oceans, lakes, grasslands, and mangroves. Oceans generally have the highest productivity due to the abundance of marine life and nutrient availability. Lakes also have high productivity. Grasslands have moderate productivity, while mangroves have relatively lower productivity due to their challenging saline and waterlogged conditions.

24. Contour bunding is a method of soil conservation used in

(a) desert margins, liable to strong wind action
(b) low flat plains, close to stream courses, liable to flooding
(c) scrublands, liable to the spread of weed growth
(d) None of the above

Answer: (b) low flat plains, close to stream courses, liable to flooding

Explanation: Contour bunding is a method of soil conservation used in low flat plains, close to stream courses, and areas that are liable to flooding. It involves the construction of bunds (small ridges) along the contours of the land to prevent soil erosion and retain water during heavy rainfall, especially in areas prone to flooding.

25. The Government enacted the Panchayat Extension to Scheduled Areas (PESA) Act in 1996. Which one of the following is not identified as its objective?

(a) To provide self-governance
(b) To recognize traditional rights
(c) To create autonomous regions in tribal areas
(d) To free tribal people from exploitation

Answer: (c) To create autonomous regions in tribal areas

Explanation: The Panchayat Extension to Scheduled Areas (PESA) Act, 1996, aims to provide self-governance to tribal communities, recognize their traditional rights over resources, and protect their culture and customs. However, it does not seek to create autonomous regions in tribal areas.

26. Under the Scheduled Tribes and Other Traditional Forest Dwellers (Recognition of Forest Rights) Act, 2006, who shall be the authority to initiate the process for determining the nature and extent of individual or community forest rights or both?

(a) State Forest Department
(b) District Collector/Deputy Commissioner
(c) Tahsildar /Block Development Officer / Mandai Revenue Officer
(d) Gram Sabha

Answer: (d) Gram Sabha

Explanation: Under the Scheduled Tribes and Other Traditional Forest Dwellers (Recognition of Forest Rights) Act, 2006, the authority to initiate the process for determining the nature and extent of individual or community forest rights or both is the Gram Sabha (village council). The Gram Sabha plays a crucial role in the recognition and vesting of forest rights to eligible forest-dwelling communities.

27. Improper handling and storage of cereal grains and oilseeds result in the production of toxins known as aflatoxins which are not generally destroyed by the normal cooking process. Aflatoxins are produced by

(a) bacteria
(b) protozoa
(c) moulds
(d) viruses

Answer: (c) moulds

Explanation: Aflatoxins are toxins produced by certain species of moulds, particularly Aspergillus flavus and Aspergillus parasiticus. These moulds can contaminate various food commodities, especially cereal grains, oilseeds, nuts, and spices. Aflatoxins are carcinogenic and harmful to human health, and they are not generally destroyed by normal cooking processes.

28. ‘Economic Justice’ the objectives of the Constitution has been provided in

(a) the Preamble and Fundamental Rights
(b) the Preamble and the Directive Principles of State Policy
(c) the Fundamental Rights and the Directive Principles of State Policy
(d) None of the above

Answer: (b) the Preamble and the Directive Principles of State Policy

Explanation: The objective of ‘Economic Justice’ is explicitly provided in the Preamble and the Directive Principles of State Policy in the Indian Constitution. The Preamble sets out the ideals and objectives of the Constitution, and it includes the goal of securing economic justice among other principles. The Directive Principles of State Policy (DPSPs) are enshrined in Part IV of the Constitution and provide guidelines for the government to promote the welfare of the people and establish economic and social justice.

29. Due to improper / indiscriminate disposal of old and used computers or their parts, which of the following are released into the environment as e-waste?

  1. Beryllium
  2. Cadmium
  3. Chromium
  4. Heptachlor
  5. Mercury
  6. Lead
  7. Plutonium

Select the correct answer using the codes given below.

(a) 1, 3, 4, 6 and 7 only
(b) 1, 2, 3, 5 and 6 only
(c) 2, 4, 5 and 7 only
(d) 1, 2, 3, 4, 5, 6 and 7

Answer: (b) 1, 2, 3, 5 and 6 only

Explanation: Due to improper or indiscriminate disposal of old and used computers or their parts, various hazardous substances are released into the environment as e-waste. The substances released include:

  1. Beryllium (Be)
  2. Cadmium (Cd)
  3. Chromium (Cr)
  4. Heptachlor is not related to e-waste and is not released from computers.
  5. Mercury (Hg)
  6. Lead (Pb)
  7. Plutonium (Pu) is not commonly found in consumer electronics and is not released as e-waste.

30. Acid rain is caused by the pollution of the environment by

(a) carbon dioxide and nitrogen
(b) carbon monoxide and carbon dioxide
(c) ozone and carbon dioxide
(d) nitrous oxide and sulphur dioxide

Answer: (d) nitrous oxide and sulphur dioxide

Explanation: Acid rain is caused by the presence of certain pollutants in the atmosphere, primarily sulphur dioxide (SO2) and nitrogen oxides (NOx). When these pollutants are released into the air from sources such as burning fossil fuels (e.g., coal and oil), industrial processes, and vehicle emissions, they react with water vapor and other atmospheric components to form sulphuric acid (H2SO4) and nitric acid (HNO3), respectively. These acids can then fall to the ground as acid rain, which can have harmful effects on the environment and ecosystems.

31. With reference to food chains in ecosystems, consider the following statements:

  1. A food chain illustrates the order in which a chain of organisms feed upon each other.
  2. Food chains are found within the populations of a species.
  3. A food chain illustrates the numbers of each organism that is eaten by others.

Which of the statements given above is/are correct?

(a) 1 only
(b) 1 and 2 only
(c) 1, 2 and 3
(d) None

Answer: (a) 1 only

Explanation: Statement 1 is correct. A food chain is a linear sequence that shows the order in which organisms feed upon each other, starting with a primary producer (e.g., plants) and ending with a top predator (e.g., apex carnivores). Statement 2 is incorrect as food chains involve different species in a community, not just within the populations of a single species. Statement 3 is also incorrect as food chains do not represent the numbers of each organism; they only show the feeding relationships.

32. Consider the following pairs

National Park – River flowing through the Park

  1. Corbett National Park – Ganga
  2. Kaziranga National Park – Manas
  3. Silent Valley National Park – Kaveri

Which of the above pairs is/are correctly matched?

(a) 1 and 2
(b) 3 only
(c) 1 and 3
(d) None

Answer: (b) 3 only

Explanation: The pair “Silent Valley National Park – Kaveri” is correctly matched. Silent Valley National Park is situated in Kerala, and the Kaveri River does flow near the park. However, the other two pairs are not correctly matched. Corbett National Park is located in Uttarakhand and does not have the Ganga flowing through it; it is primarily associated with the Ramganga River. Kaziranga National Park is in Assam and is not connected to the Manas River.

33. Consider the following organisms

  1. Agaricus
  2. Nostoc
  3. Spirogyra

Which of the above is/are used as bio-fertilizer/bio-fertilizers?

(a) 1 and 2
(b) 2 only
(c) 2 and 3
(d) 3 only

Answer: (c) 2 and 3

Explanation: Nostoc and Spirogyra are used as bio-fertilizers. Agaricus is not used as a bio-fertilizer. It is a type of mushroom that is edible and commercially cultivated for food.

34. Which of the following adds/add nitrogen to the soil?

  1. Excretion of urea by animals
  2. Burning of coal by man
  3. Death of vegetation

Select the correct answer using the codes given below.

(a) 1 only
(b) 2 and 3 only
(e) 1 and 3 only
(d) 1, 2 and 3

Answer: (c) 1 and 3 only

Explanation:

  1. Excretion of urea by animals: Urea is a nitrogenous waste product excreted by animals, and it decomposes into ammonia, which can be converted into nitrate by nitrifying bacteria. This process adds nitrogen to the soil.
  2. Burning of coal by man: Burning of coal releases nitrogen oxides into the atmosphere, but it does not directly add nitrogen to the soil.
  3. Death of vegetation: When vegetation dies and decomposes, it releases organic matter and nitrogen back into the soil, enriching it with nitrogen.

35. In which of the following States is the lion-tailed macaque found in its natural habitat?

  1. Tamil Nadu
  2. Kerala
  3. Karnataka
  4. Andhra Pradesh

Select the correct answer using the codes given below.

(a) 1, 2 and 3 only
(b) 2 only
(c) 1, 3 and 4 only
(d) 1, 2, 3 and 4

Answer: (b) 2 only

Explanation: The lion-tailed macaque (Macaca silenus) is found in its natural habitat in the Western Ghats of India. It is an endangered species and is primarily found in the tropical rainforests of the states of Kerala, particularly in the Silent Valley National Park, and parts of Karnataka.

36. Some Buddhist rock-cut caves are called Chaityas, while the others are called Viharas. What is the difference between the two?

(a) Vihara is a place of worship, while Chaitya is the dwelling place of the monks
(b) Chaitya is a place of worship, while Vihara is the dwelling place of the monks
(c) Chaitya is the stupa at the far end of the cave, while Vihara is the hall axial to it
(d) There is no material difference between the two

Answer: (b) Chaitya is a place of worship, while Vihara is the dwelling place of the monks

Explanation: The main difference between Chaityas and Viharas in Buddhist rock-cut caves lies in their purposes and functions:

– Chaitya: A Chaitya is a place of worship in Buddhism. It typically contains a stupa, which is a dome-like structure housing relics or representations of the Buddha. It serves as a sacred space where Buddhist monks and devotees gather for religious ceremonies, meditation, and prayer.

– Vihara: A Vihara, on the other hand, is a dwelling place or monastery for Buddhist monks. It is a residential space where monks live and engage in meditation, study, and other monastic activities. Viharas provide a supportive environment for the monks’ spiritual practices and daily routines.

37. Which one of the following describes best the concept of Nirvana in Buddhism?

(a) The extinction of the flame of desire
(b) The complete annihilation of self
(c) A state of bliss and rest
(d) A mental stage beyond all comprehension

Answer: (a) The extinction of the flame of desire

Explanation: The concept of Nirvana in Buddhism refers to the ultimate goal of liberation and spiritual awakening. It is often described as the “extinction of the flame of desire,” meaning the complete cessation of craving, attachment, and suffering.

Nirvana is a state of profound peace, freedom from the cycle of birth and death (samsara), and the end of the individual’s sense of self (ego). It is the state of enlightenment achieved by Buddhas and advanced practitioners who have overcome ignorance and attained liberation from the cycle of suffering.

38. According to the Constitution of India, which of the following are fundamental for the governance of the country?

(a) Fundamental Rights
(b) Fundamental Duties
(c) Directive Principles of State Policy
(d) Fundamental Rights and Fundamental Duties

Answer: (a) Fundamental Rights

Explanation: According to the Constitution of India, Fundamental Rights are considered fundamental for the governance of the country. They are enshrined in Part III of the Constitution (Articles 12 to 35) and guarantee certain basic rights and freedoms to the citizens of India, ensuring their dignity, equality, and protection from discrimination. Fundamental Rights are justiciable, which means they can be enforced through the courts if violated.

39. The people of India agitated against the arrival of the Simon Commission because

(a) Indians never wanted the review of the working of the Act of 1919
(b) Simon Commission recommended the abolition of Dyarchy (Diarchy) in the Provinces
(c) there was no Indian member in the Simon Commission
(d) the Simon Commission suggested the partition of the country

Answer: (c) There was no Indian member in the Simon Commission

Explanation: The people of India agitated against the arrival of the Simon Commission primarily because there was no Indian member included in the Commission. The Simon Commission, appointed in 1927, was intended to review and recommend constitutional reforms in British India. However, it consisted only of British members, with no representation from the Indian National Congress or other Indian political parties. This exclusion of Indian members led to widespread protests and demands for Indian representation in the decision-making process. The Indian National Congress boycotted the Simon Commission, and it marked a significant turning point in the Indian freedom struggle.

40. Quit India Movement was launched in response to

(a) Cabinet Mission Plan
(b) Cripps Proposals
(c) Simon Commission Report
(d) Wavell Plan

Answer: (b) Cripps Proposals

Explanation: The Quit India Movement was launched in response to the Cripps Proposals, which were put forward by Sir Stafford Cripps, a British cabinet minister, in March 1942. The Cripps Proposals offered limited autonomy and a promise of post-war dominion status to India, subject to certain conditions.

However, the proposals fell short of meeting Indian demands for immediate and complete independence. Consequently, the Indian National Congress led by Mahatma Gandhi rejected the proposals and launched the Quit India Movement on August 9, 1942. The movement called for the immediate end of British rule in India and resulted in widespread civil disobedience and mass protests across the country.

41. The balance of payments of a country is a systematic record of

(a) all import and transactions of a during a given period normally a year
(b) goods exported from a country during a year
(c) economic transaction between the government of one country to another
(d) capital movements from one country to another

Answer: (a) all import and transactions of a during a given period, normally a year

Explanation: The balance of payments (BoP) of a country is a systematic record of all economic transactions between the residents of that country and the rest of the world during a specific period, typically a year. It includes records of exports and imports of goods and services, payments for investments and loans, and other financial transactions between the country and other nations. The BoP is divided into two main accounts: the current account (for trade in goods and services) and the capital account (for financial and investment transactions).

42. The Reserve Bank of India regulates the commercial banks in matters of

  1. liquidity of assets
  2. branch expansion
  3. merger of banks
  4. winding-up of banks

Select the correct answer using the codes given below.

(a) 1 and 4 only
(b) 2, 3 and 4 only
(c) 1, 2 and 3 only
(d) 1, 2, 3 and 4

Answer: (a) 1 and 4 only

Explanation: The Reserve Bank of India (RBI) regulates commercial banks in India on matters of liquidity of assets and winding-up of banks.

1. Liquidity of assets: The RBI sets certain guidelines for the banks to maintain a certain level of liquidity of their assets, ensuring that they have enough liquid funds to meet their obligations and withdrawals by depositors. This is done to maintain financial stability and prevent banking crises.

2. Branch expansion: The RBI does not directly regulate the branch expansion of commercial banks. However, it can issue guidelines and permissions for banks to open new branches to ensure that banking services are available to underserved areas and promote financial inclusion.

3. Merger of banks: The RBI plays a significant role in overseeing and approving the merger of banks in India. It evaluates and approves merger proposals based on various factors, such as the financial health of the banks involved and the potential benefits to the banking sector and the economy.

4. Winding-up of banks: The RBI has the authority to initiate the process of winding up or liquidation of a bank in case of severe financial distress or insolvency. This measure is taken to protect the interests of depositors and maintain financial stability in the banking system.**

43. An increase in the Bank Rate generally indicates that the

(a) market rate of interest is likely to fall
(b) Central Bank is no longer making loans to commercial banks
(c) Central Bank is following an easy money policy
(d) Central Bank is following a tight money policy

Answer: (d) Central Bank is following a tight money policy

Explanation: An increase in the Bank Rate generally indicates that the Central Bank is following a tight money policy.

The Bank Rate, also known as the discount rate or policy rate, is the rate at which the Central Bank (in India, the Reserve Bank of India) lends to commercial banks in the event of a shortage of funds. When the Central Bank increases the Bank Rate, it makes borrowing from the Central Bank more expensive for commercial banks. As a result, commercial banks are discouraged from borrowing, and they, in turn, pass on the increased costs to borrowers by raising their lending rates. This tighter credit policy aims to control inflation, reduce demand, and restrict the supply of money in the economy. It is used as a monetary policy tool to combat inflationary pressures and stabilize the economy.

44. In India, deficit financing is used for raising resources for

(a) economic development
(b) redemption of public debt
(c) adjusting the balance of payments
(d) reducing the foreign debt

Answer: (a) economic development

Explanation: Deficit financing in India is primarily used for raising resources for economic development.

Deficit financing is a method through which the government meets its budget deficit by creating additional money or borrowing from the Central Bank or other sources. When the government’s total expenditures exceed its total revenues, it results in a budget deficit.

Deficit financing is used to bridge this gap and fund various development projects and government programs aimed at promoting economic growth, infrastructure development, social welfare, and other developmental activities. By using deficit financing, the government can raise funds to invest in projects that contribute to economic development and address the needs of the country’s growing population. However, deficit financing should be managed carefully to ensure that it does not lead to inflationary pressures and unsustainable levels of debt.

45. Which of the following characterizes/ characterize the people of the Indus Civilization?

  1. They possessed great palaces and temples.
  2. They worshipped both male and female deities.
  3. They employed horse-drawn chariots in warfare.

Select the correct statement/ statements using the codes given below.

(a) 1 and 2 only
(b) 2 only
(e) 1, 2 and 3
(d) None of the statements given above is correct

Answer: (b) None of the statements given above is correct

Explanation:

1. They possessed great palaces and temples: The Indus Valley Civilization, also known as the Harappan Civilization, did not feature monumental architecture in the form of palaces and temples like some other ancient civilizations such as Mesopotamia or Egypt. The architecture of the Indus cities primarily consisted of well-planned and organized urban centers with well-constructed residential and public buildings.

2. They worshipped both male and female deities: The religious practices of the Indus Valley Civilization are not fully deciphered, and the specific details of their deities and religious beliefs are not well-known. Archaeological evidence from the civilization indicates the presence of religious symbols and artifacts, they used to worship both male and female deities.

3. They employed horse-drawn chariots in warfare: There is no conclusive evidence to suggest that the people of the Indus Valley Civilization used horse-drawn chariots in warfare or other activities.

46. Which of the following diseases can be transmitted from one person to another through tattooing?

1. Chikungunya
2. Hepatitis B
3. HIV-AIDS

Select the correct answer using the codes given below.

(a) 1 and 2 only
(b) 2 only
(e) 1, 2 and 3
(d) None of the statements given above is correct

Answer: (b) 2 only

Explanation:

  • Chikungunya: Chikungunya is a viral disease transmitted primarily through the bite of infected mosquitoes, such as Aedes aegypti and Aedes albopictus. It is not transmitted through tattooing or direct contact with infected individuals.
  • Hepatitis B: Hepatitis B is a viral infection that affects the liver and is transmitted through contact with infected blood, body fluids, or through sexual contact. Tattooing using unsterilized or contaminated equipment, such as needles or ink, can pose a risk of transmitting hepatitis B if the equipment is contaminated with infected blood.
  • HIV-AIDS: HIV (Human Immunodeficiency Virus) is the virus that causes AIDS (Acquired Immunodeficiency Syndrome). HIV is primarily transmitted through sexual contact, sharing of needles by drug users, or from an infected mother to her child during childbirth or breastfeeding. While there have been some concerns about the potential risk of HIV transmission through tattooing, there is no confirmed case of HIV transmission solely through the tattooing process. However, using unsterilized equipment can increase the risk of transmission if contaminated with infected blood.

It is essential to ensure that proper infection control practices, including the use of sterile equipment, are followed during tattooing to minimize the risk of infections, including hepatitis B and other blood-borne diseases.

47. Which of the following statements is/are applicable to Jain doctrine?

  1. The surest way of annihilating Karma is to practice penance.
  2. Every object, even the smallest particle, has a soul.
  3. Karma is the bane of the soul and must be ended.

Select the correct answer using the codes given below.

(a) 1 only
(b) 2 and 3 only
(c) 1 and 3 only
(d) 1, 2 and 3

Answer: (c) 1 and 3 only

Explanation:

  1. The surest way of annihilating Karma is to practice penance: This statement is applicable to Jain doctrine. Jainism emphasizes the importance of self-discipline and penance to purify the soul and liberate it from the bondage of karma. The practice of penance (austerities) is seen as a means to reduce and eventually eradicate accumulated karmas.
  2. Every object, even the smallest particle, has a soul: This statement is not specific to Jain doctrine. Jainism indeed teaches the concept of anekantavada, which recognizes multiple perspectives and the relativity of truth, but it does not directly assert that every object, no matter how small, has a soul. Instead, Jainism emphasizes that all living beings have souls, and this includes not only humans and animals but also plants and microorganisms.
  3. Karma is the bane of the soul and must be ended: This statement is applicable to Jain doctrine. Jainism teaches that karma is the cause of bondage and suffering for the soul. To attain liberation (moksha), one must strive to end the inflow of karma by following the path of right belief, right knowledge, and right conduct (triratna).

48. Which one of the following terms describes not only the physical space occupied by an organism but also its functional role in the community of organisms?

(a) Ecotone
(b) Ecological niche
(c) Habitat
(d) Home range

Answer: (b) Ecological niche

Explanation:

  • Ecological niche refers to the role and position of an organism within an ecosystem. It includes not only the physical space (habitat) occupied by the organism but also its interactions with other organisms, its use of resources, and its functional role in the community of organisms. The ecological niche of an organism can be thought of as its “profession” or “job” within the ecosystem, as it describes how the organism meets its needs for survival and reproduction.
  • Ecotone is the transition zone between two different ecosystems or habitats, where the characteristics of both ecosystems intermingle.
  • Habitat refers to the specific physical location or environment where an organism lives and can find the resources needed for survival.
  • Home range refers to the area that an animal or a group of animals regularly use for their activities, including foraging and mating, within their larger habitat.

49. Photochemical smog is a resultant of the reaction among

(a) NO2, 03 and peroxyacetyl nitrate in the presence of sunlight
(b) CO, 02 and peroxyacetyl nitrate in the presence of sunlight
(c) CO, CO2 and N02 at low temperature
(d) High concentration of N02, O3 and CO in the evening

Answer: (a) NO2, O3, and peroxyacetyl nitrate in the presence of sunlight

Explanation:

  • Photochemical smog is a type of smog that is formed when certain air pollutants react with sunlight. It is most common in urban areas with high traffic and industrial emissions.
  • The primary pollutants involved in the formation of photochemical smog are nitrogen dioxide (NO2) and volatile organic compounds (VOCs) emitted from vehicles and industrial sources.
  • In the presence of sunlight, nitrogen dioxide (NO2) undergoes a photochemical reaction, leading to the formation of ozone (O3). Ozone is a major component of photochemical smog and contributes to its characteristic brownish color.
  • Another important component of photochemical smog is peroxyacetyl nitrate (PAN), which is formed from the reaction of NO2 and VOCs in the presence of sunlight.
  • The reaction can be summarized as follows: NO2 + sunlight → NO + O O + O2 → O3 (ozone) NO2 + VOCs + sunlight → PAN + other oxidants

50. Consider the following minerals

  1. Calcium
  2. Iron
  3. Sodium

Which of the minerals given above is/are required by the human body for the contraction of muscles?

(a) 1 only
(b) 2 and 3 only
(c) 1 and 3 only
(d) 1, 2 and 3

Answer: (a) 1 only

Explanation:

Calcium is required by the human body for the contraction of muscles. Calcium ions play a crucial role in muscle contraction as they are involved in the interaction between actin and myosin, which are proteins responsible for muscle contraction. When a muscle receives a signal from the nervous system to contract, calcium ions are released from storage in the muscle cells and bind to specific proteins, enabling the actin and myosin filaments to slide past each other, leading to muscle contraction.

Iron is important for various physiological processes in the human body, but it is not directly involved in muscle contraction. Iron is primarily involved in the transport of oxygen in the blood, as it is a component of hemoglobin in red blood cells.

Sodium is important for maintaining the body’s fluid balance and nerve function but is not directly involved in muscle contraction.

51. Consider the following statements: The Parliamentary Committee on Public Accounts

  1. Consists of not more than 25 Members of the Lok Sabha.
  2. Scrutinizes appropriation finance accounts of the Government.
  3. The Auditor General examines the report Comptroller and General of India.

Which of the statements given above is/are correct?

(a) 1 only
(b) 2 and 3 only
(e) 3 only
(d) 1, 2 and 3

Answer: (c) 3 only

Explanation:

  • The statements are partially correct. The Parliamentary Committee on Public Accounts is a committee of the Parliament of India that primarily examines the audit reports of the Comptroller and Auditor General of India (CAG). However, the other statements are not accurate.
  • Statement 1: The committee consists of Members of Parliament (MPs) from both the Lok Sabha and the Rajya Sabha. It is not limited to only 25 Members of the Lok Sabha.
  • Statement 2: The committee examines the reports of the CAG related to the audit of government expenditures, including the appropriation animals, and other organisms. While some viruses can cause diseases and infections in living organisms, they do not play a role in the decomposition of dead organic matter in ecosystems.

52. Consider the following Bhakti Saints:

1. Dadu Dayal
2. Guru Nanak
3. Tyagaraja

Who among the above was/were preaching when the Lodi dynasty fell and Babur took over?

(a) 1 and 3
(b) 2 only
(c) 2 and 3
(d) 1 and 2

(b) 2 only

Explanation: Guru Nanak was the Bhakti Saint who was preaching when the Lodi dynasty fell and Babur took over. Guru Nanak was the founder of Sikhism and was born in 1469. He preached the message of monotheism, equality, and social justice. His teachings laid the foundation for the Sikh faith and community. The Lodi dynasty fell in 1526 after the Battle of Panipat, and Babur, the founder of the Mughal Empire, became the ruler of Delhi and the northern Indian plains.

53. With reference to the food chains in ecosystems, which of the following kinds of organism is / are known as decomposer organism/organisms?

1. Virus
2. Fungi
3. Bacteria

Select the correct answer using the codes given below.

(a) 1 only
(b) 2 and 3 only
(c) 1 and 3 only
(d) 1, 2 and 3

(b) 2 and 3 only

Explanation: Decomposer organisms are responsible for breaking down organic matter and dead bodies of plants and animals into simpler substances. They play a crucial role in recycling nutrients back into the ecosystem. Among the given options:

  1. Viruses are not considered decomposer organisms as they cannot independently carry out metabolic processes and require a host cell for replication. They are not capable of breaking down organic matter like fungi and bacteria.
  2. Fungi and 3. Bacteria are both decomposer organisms. They secrete enzymes that help them break down complex organic materials into simpler substances, which are then absorbed by them for nutrition. These simpler substances are eventually returned to the soil, making them available for other organisms in the food chain.

Therefore, the correct answer is option (b) 2 and 3 only.

54. The most important fishing grounds of the world are found in the regions where

(a) warm and cold atmospheric currents meet
(b) rivers drain out large amounts of freshwater into the sea
(c) warm and cold oceanic currents meet
(d) the continental shelf is undulating

Answer: (c) warm and cold oceanic currents meet

Explanation:

  • The most important fishing grounds in the world are generally found in areas where warm and cold oceanic currents meet. These regions are characterized by high biological productivity due to the mixing of nutrient-rich cold waters with nutrient-poor warm waters. The mixing of currents creates favorable conditions for the growth of plankton, which forms the base of the marine food chain and attracts a diverse range of fish and other marine species.
  • Nutrient-rich cold currents from deeper ocean layers bring up nutrients such as nitrates and phosphates to the surface, supporting the growth of phytoplankton (microscopic algae). In turn, phytoplankton provides food for zooplankton and other marine organisms, leading to a concentration of marine life in these areas.
  • The meeting of warm and cold currents creates a dynamic and diverse marine environment, attracting various fish species that seek these nutrient-rich waters for feeding and spawning. Fishing activities in these regions are highly productive and significant for global fishery industries.

55. Which of the following is/are unique characteristic/characteristics of equatorial forests?

  1. Presence of tall, closely set trees with crowns forming a continuous canopy.
  2. Coexistence of a large number of species.
  3. Presence of numerous varieties of epiphytes.

Select the correct answer using the code given below:

(a) 1 only
(b) 2 and 3 only
(c) 1 and 3 only
(d) 1, 2, and 3

Answer: (d) 1, 2, and 3

Explanation:

  • Equatorial forests, also known as tropical rainforests, are found near the equator in regions with high rainfall and constant temperatures throughout the year. They are characterized by a unique combination of features, making them one of the most biodiverse and productive ecosystems on Earth.
  1. Presence of tall, closely set trees with crowns forming a continuous canopy: This is a characteristic feature of equatorial forests. The dense vegetation consists of tall trees that grow closely together and have broad, overlapping crowns. The continuous canopy formed by the tree crowns blocks most of the sunlight from reaching the forest floor, creating a shaded and humid environment below.
  2. Coexistence of a large number of species: Equatorial forests are renowned for their exceptional biodiversity. They support an incredibly diverse array of plant and animal species, including various types of trees, plants, insects, birds, and mammals. The high species richness is attributed to the stable and favorable climatic conditions throughout the year, which allow a wide range of species to thrive.
  3. Presence of numerous varieties of epiphytes: Epiphytes are plants that grow on the branches and trunks of other plants without drawing nutrients from the host plant. Equatorial forests are home to a wide variety of epiphytic plants, such as mosses, ferns, and orchids. These plants have adaptations that allow them to absorb moisture and nutrients from the humid air and rainwater, rather than from the soil.

56. Which of the following constitute the Capital Account?

  1. Foreign Loans
  2. Foreign Direct Investment
  3. Private Remittances
  4. Portfolio Investment

Select the correct answer using the codes given below.

(a) 1, 2, and 3
(b) 1, 2, and 4
(c) 2, 3, and 4
(d) 1, 3, and 4

Answer: (b) 1, 2, and 4

Explanation:

  • The Capital Account is one of the two primary components of a country’s balance of payments, with the other being the Current Account. The Capital Account records the flow of capital into and out of a country and includes the following components:
  1. Foreign Loans: This includes loans obtained by the government and private sector from foreign sources. Foreign loans may be long-term loans, short-term loans, or commercial borrowings.
  2. Foreign Direct Investment (FDI): FDI refers to the investment made by foreign entities (individuals or companies) in the productive assets of a country, such as the establishment of factories, infrastructure projects, and the acquisition of companies.
  3. Private Remittances: Private remittances refer to the money sent by individuals working abroad (migrant workers) back to their home country to support their families or for other purposes.
  4. Portfolio Investment: Portfolio investment includes investments in financial assets, such as stocks, bonds, and other securities, by foreign investors in a country’s financial markets.

57. Consider the following historical places:

  1. Ajanta Caves
  2. Lepakshi Temple
  3. Sanchi Stupa

Which of the above places is/are also known for mural paintings?

(a) 1 only
(b) 1 and 2 only
(c) 1, 2, and 3
(d) None

Answer: (a) 1 only

Explanation:

  • Ajanta Caves: Ajanta Caves, located in Maharashtra, India, are famous for their exquisite mural paintings. These caves date back to the 2nd century BCE and 6th century CE and are considered masterpieces of Buddhist religious art. The paintings depict various scenes from the life of Buddha, Jataka tales, and other mythological stories.
  • Lepakshi Temple: The Lepakshi Temple in Andhra Pradesh, India, is known for its stunning architecture and intricate carvings. While it is renowned for its artistic beauty, it is not particularly famous for mural paintings.
  • Sanchi Stupa: Sanchi Stupa, located in Madhya Pradesh, India, is a UNESCO World Heritage Site and one of the oldest stone structures in India. It is an important Buddhist monument and a significant pilgrimage site. While Sanchi Stupa is adorned with intricate stone carvings, it does not feature mural paintings.
  • Conclusion: Therefore, only the Ajanta Caves (1) among the given historical places are known for mural paintings. The correct answer is (a) 1 only.

58. With reference to the history of philosophical thought in India, consider the following statements regarding the Sankhya school:

  1. Sankhya does not accept the theory of rebirth or transmigration of the soul.
  2. Sankhya holds that it is self-knowledge that leads to liberation and not any exterior influence or agent.

Which of the statements given above is/are correct?

(a) 1 only
(b) 2 only
(c) Both 1 and 2
(d) Neither 1 nor 2

Answer: (c) Both 1 and 2

Explanation:

  1. Sankhya does not accept the theory of rebirth or transmigration of the soul: This statement is correct. The Sankhya school of philosophy, founded by Sage Kapila, is one of the six classical schools of Indian philosophy. Sankhya philosophy is dualistic and considers the material world (prakriti) and the eternal, individual souls (purushas). According to Sankhya, the individual souls are distinct and do not undergo a cycle of rebirth or transmigration (samsara). Liberation (moksha) in Sankhya is achieved through knowledge and understanding of the nature of prakriti and purusha.
  2. Sankhya holds that it is self-knowledge that leads to liberation and not any exterior influence or agent: This statement is also correct. In Sankhya philosophy, liberation (moksha) is attained through self-knowledge (atma-jnana) or the realization of the true nature of the self (purusha) as distinct from prakriti (matter). The process of liberation involves understanding the distinction between the eternal, unchanging self (purusha) and the ever-changing material world (prakriti). Sankhya emphasizes the path of knowledge (jnana yoga) to achieve liberation.

59. In the context of India, which of the following principles is/are implied institutionally in the parliamentary government?

  1. Members of the Cabinet are Members of the Parliament.
  2. Ministers hold the office till they enjoy confidence in the Parliament.
  3. The Cabinet is headed by the Head of the State.

Select the correct answer using the codes given below.

(a) 1 and 2 only
(b) 3 only
(c) 2 and 3 only
(d) 1, 2, and 3

Answer: (a) 1 and 2 only

Explanation:

  • The parliamentary form of government is a system where the executive branch (Cabinet) derives its legitimacy from and is accountable to the legislative branch (Parliament). In the context of India, the parliamentary government is based on the principles mentioned in the Constitution of India.
  1. Members of the Cabinet are Members of the Parliament: This principle is applicable in India’s parliamentary government. According to the Constitution of India, most members of the Council of Ministers (Cabinet) are typically chosen from among the elected members of the Parliament (Lok Sabha and Rajya Sabha). This ensures that the executive branch is directly accountable to the legislative branch and reflects the will of the people.
  2. Ministers hold the office till they enjoy confidence in the Parliament: This principle is also applicable in India’s parliamentary government. The Council of Ministers (Cabinet) is collectively responsible to the Parliament. The Cabinet holds office as long as it enjoys the confidence of the majority of members in the Lok Sabha. If the Cabinet loses the confidence of the Parliament, it must resign, and the President of India may dissolve the Lok Sabha.
  3. The Cabinet is headed by the Head of the State: This statement is not correct. In India’s parliamentary system, the Cabinet is headed by the Prime Minister, who is the head of the government. The President of India, on the other hand, is the head of state but does not play an active role in day-to-day governance. The President’s role is largely ceremonial and symbolic.
  • Conclusion: Therefore, the correct answer is (a) 1 and 2 only.

60. The annual range of temperature in the interior of the continents is high as compared to coastal areas. What is/are the reason/reasons?

  1. Thermal difference between land and water.
  2. Variation in altitude between continents and oceans.
  3. Presence of strong winds in the interior.
  4. Heavy rains in the interior as compared to coasts.

Select the correct answer using the codes given below.

(a) 1 only
(b) 1 and 2 only
(c) 2 and 3 only
(d) 1, 2, 3, and 4

Answer: (a) 1 only

Explanation: The reason for the high annual range of temperature in the interior of the continents as compared to coastal areas is primarily due to “Thermal difference between land and water.”

  1. Thermal Difference Between Land and Water: Land heats up and cools down faster than water. In the interior of the continents, there is a greater expanse of land and a lesser expanse of water bodies such as oceans or seas. During the day, the land heats up rapidly, and during the night, it cools down quickly. This results in a significant difference between the day and night temperatures, leading to a high annual temperature range.
  2. Variation in Altitude Between Continents and Oceans: While variation in altitude can influence temperature differences, it is not the primary reason for the high annual temperature range in the interior of the continents.
  3. Presence of Strong Winds in the Interior: Strong winds can influence temperature variations to some extent, but they are not the main reason for the high annual temperature range in the interior of the continents.
  4. Heavy Rains in the Interior as Compared to Coasts: The presence of heavy rains in the interior is not directly related to the high annual temperature range. Rainfall patterns are more related to factors like prevailing wind patterns and proximity to water bodies.

Therefore, the correct answer is option (a) 1 only.

61. Which of the following is/are the characteristic/characteristics of Indian coal?
  1. High ash content
  2. Low sulfur content
  3. Low ash fusion temperature

Select the correct answer using the codes given below.

(a) 1 and 2 only
(b) 2 only
(c) 1 and 3 only
(d) 1, 2 and 3

Answer: (a) 1 and 2 only

Explanation:

  • Indian coal is known for its specific characteristics, which can vary depending on the region and type of coal. The given characteristics are as follows:
  1. High ash content: Indian coal is known for its relatively high ash content. Ash is the non-combustible mineral residue left after burning coal. The presence of high ash content in coal affects its calorific value and combustion efficiency.
  2. Low sulfur content: One of the positive aspects of Indian coal is that it generally has a low sulfur content. Low sulfur coal is considered environmentally favorable because it results in lower emissions of sulfur dioxide (SO2) during combustion, which is a major contributor to air pollution.
  3. Low ash fusion temperature: The statement mentioning “low ash fusion temperature” is incorrect. In reality, Indian coal tends to have a high ash fusion temperature. Ash fusion temperature is the temperature at which the ash in coal begins to melt and form clinkers. High ash fusion temperature is beneficial because it reduces the likelihood of clinker formation, which can cause operational issues in boilers and furnaces.

62. Which of the following statements regarding laterite soils of India are correct?

  1. They are generally red in color.
  2. They are rich in nitrogen and potash.
  3. They are well-developed in Rajasthan and UP.
  4. Tapioca and cashew nuts grow well on these soils.

Select the correct answer using the codes given below.

(a) 1, 2 and 3
(b) 2, 3 and 4
(c) 1 and 4
(d) 2 and 3 only

Answer: (c) 1 and 4

Explanation:

  • Laterite soils are a type of soil found in tropical regions with high rainfall and high temperatures. In India, laterite soils are primarily found in the western coastal region, northeastern states, parts of central India, and some areas of the Deccan Plateau.
  1. They are generally red in color: This statement is correct. Laterite soils are reddish-brown to deep red in color due to the presence of iron and aluminum oxides. The iron oxide (hematite) gives the soil its characteristic reddish hue.
  2. They are rich in nitrogen and potash: This statement is incorrect. Laterite soils are typically deficient in nitrogen and potash. They are leached soils and have lost most of their nutrients due to heavy rainfall and weathering processes.
  3. They are well-developed in Rajasthan and UP: This statement is incorrect. Laterite soils are not well-developed in Rajasthan and Uttar Pradesh. They are more commonly found in regions with high rainfall and high humidity.
  4. Tapioca and cashew nuts grow well on these soils: This statement is correct. Despite their nutrient deficiencies, laterite soils are used for growing certain crops that are adapted to such conditions. Tapioca (cassava) and cashew nuts are examples of crops that can grow well in laterite soils.

63. Consider the following statements:

  1. Natural gas occurs in the Gondwana beds.
  2. Mica occurs in abundance in Kodarma.
  3. Dharwars are famous for petroleum.

Which of the statements given above is/are correct?

(a) 1 and 2
(b) 2 only
(c) 2 and 3
(d) None

Answer: (d) None

Explanation:

  • Natural gas is primarily found in association with sedimentary rocks, especially in the sedimentary basins. It is often found in reservoirs trapped within porous rock formations. The statement that natural gas occurs in the Gondwana beds is not correct. The Gondwana beds are known for their coal deposits and not for natural gas.
  • Mica is a mineral used in various industrial applications, and Kodarma in Jharkhand is indeed one of the major mica-producing regions in India. Therefore, the statement about mica occurring in abundance in Kodarma is correct.
  • Dharwars are a type of geological formation characterized by hard, metamorphic rocks. They are not famous for petroleum. Petroleum (crude oil) is typically found in sedimentary rocks and not in Dharwars.

64. Consider the following crops:

  1. Cotton
  2. Groundnut
  3. Rice
  4. Wheat

Which of these are Kharif crops?

(a) 1 and 4
(b) 2 and 3 only
(c) 1, 2 and 3
(d) 2, 3 and 4

Answer: (d) 2, 3, and 4

Explanation: In India, agricultural crops are classified into two main categories based on their sowing and harvesting seasons: Kharif crops and Rabi crops.

Kharif crops are sown in the monsoon season (June to July) and harvested in the autumn (September to October). The monsoon rainfall is essential for the growth of Kharif crops. The following crops are Kharif crops:

  1. Groundnut (Peanut): Groundnut is a Kharif crop that requires warm temperatures and is grown during the monsoon season.
  2. Rice: Rice is one of the major Kharif crops, and India is one of the world’s largest rice producers.
  3. Wheat: Wheat is not a Kharif crop. It is a Rabi crop that is sown during the winter season and harvested in the spring (April to May).
  4. Cotton: Cotton is a Kharif crop, sown during the monsoon season and harvested in the autumn.

65. “Climate is extreme, rainfall is scanty, and the people used to be nomadic herders.” The above statement best describes which of the following regions?

(a) African Savannah
(b) Central Asian Steppe
(c) North American Prairie
(d) Siberian Tundra

Answer: (b) Central Asian Steppe

Explanation: The statement “Climate is extreme, rainfall is scanty, and the people used to be nomadic herders” describes the characteristics of the Central Asian Steppe.

  • Central Asian Steppe: The Central Asian Steppe is a vast semi-arid region in the heart of Asia, covering parts of countries such as Kazakhstan, Uzbekistan, Turkmenistan, and parts of Russia. It is characterized by its continental climate, with hot summers and cold winters. Rainfall in this region is limited, making it a dry and arid landscape.
  • Extreme Climate: The Central Asian Steppe experiences extreme temperature variations due to its continental location. Summers can be very hot, with temperatures soaring, and winters can be extremely cold, with temperatures dropping significantly, often below freezing.
  • Scanty Rainfall: The region receives limited rainfall, and as a result, it is relatively arid. The scarcity of water resources contributes to the arid nature of the landscape.
  • Nomadic Herders: Due to the harsh environment and limited agricultural potential, traditional livelihoods in the Central Asian Steppe have often revolved around nomadic herding practices. Nomadic communities have historically raised livestock, such as sheep, goats, and camels, which are better suited to the arid conditions of the steppe.
  • Other Options: The other options mentioned (African Savannah, North American Prairie, and Siberian Tundra) have different climate and ecological characteristics:
    • African Savannah: The African Savannah is a grassland ecosystem found in Africa, characterized by a tropical climate with distinct wet and dry seasons. It supports a diverse range of wildlife, and some regions are known for large migratory herds of animals.
    • North American Prairie: The North American Prairie is a vast grassland ecosystem in North America, characterized by fertile soils and a temperate climate. Historically, it was home to large herds of bison.
    • Siberian Tundra: The Siberian Tundra is a northernmost region of Russia characterized by extremely cold temperatures and a treeless landscape. It is known for its permafrost and limited vegetation.

66. Consider the following statements:

  1. Inflation benefits the debtors.
  2. Inflation benefits the bond-holders.

Which of the statements given above is/are correct?

(a) 1 only
(b) 2 only
(c) Both 1 and 2
(d) Neither 1 nor 2

Answer: (c) Both 1 and 2

Explanation: Inflation is the increase in the general price level of goods and services over a period of time, resulting in a decrease in the purchasing power of money. It has various effects on different economic agents, including debtors and bond-holders:

  1. Inflation benefits the debtors: This statement is correct. Debtors are individuals, businesses, or governments that have borrowed money and owe debts. Inflation can be beneficial for debtors because it reduces the real value of their debts. As the general price level rises, the nominal value of debts remains the same, but the real burden of the debt decreases. Debtors can repay their loans with money that has lower purchasing power due to inflation, effectively reducing the “real” cost of repayment.
  2. Inflation benefits the bond-holders: This statement is also correct. Bond-holders are individuals or entities that hold fixed-income securities like government or corporate bonds. Inflation erodes the purchasing power of money, which means that the future payments (interest and principal) received by bond-holders have a reduced real value. However, this reduction in the real value of future payments is already factored into the interest rate at which the bonds were issued. Bond issuers typically adjust the interest rate to compensate for expected inflation. Therefore, in a moderate inflationary environment, bond-holders can still earn a positive real rate of return on their investments.

67. Disguised unemployment generally means

(a) large number of people remain unemployed
(b) alternative employment is not available
(c) marginal productivity of labour is zero
(d) productivity of workers is low

Answer: (c) The marginal productivity of labor is zero

Explanation: Disguised unemployment refers to a situation in which more people are employed in a particular activity or sector than required for optimal productivity, and their marginal productivity is essentially zero. Disguised unemployment is a feature of underdeveloped or developing economies, especially in the agricultural sector.

  • In a situation of disguised unemployment, a large number of people are engaged in unproductive or redundant jobs, which do not contribute significantly to the overall output or productivity. It appears as if these individuals are employed, but in reality, their presence does not add any additional value to the production process.
  • The term “disguised unemployment” implies that the surplus labor is “disguised” or hidden within the system. If some of these surplus workers were removed from the activity, the overall output would remain unchanged, as their marginal productivity is zero or negligible.
  • Disguised unemployment is often found in the agricultural sector of developing countries, where the majority of the labor force is engaged in subsistence farming or low-productivity activities. Due to land fragmentation or lack of alternative employment opportunities, more people are engaged in farming than required for efficient production.
  • In such a scenario, removing a few workers from the agricultural activity would not affect the total output because the marginal productivity of labor is negligible. However, this situation also indicates underutilization of human resources and highlights the need for structural changes in the economy to create more productive and gainful employment opportunities.

68. Consider the following statements:

  1. The Council of Ministers in the Centre shall be collectively responsible to the Parliament.
  2. The Union Ministers shall hold the office during the pleasure of the President of India.
  3. The Prime Minister shall communicate to the President about the proposals for legislation.

Which of the statements given above is/are correct?

(a) 1 only
(b) 2 and 3 only
(c) 1 and 3 only
(d) 1,2 and 3

Answer: (d) 1, 2, and 3

Explanation:

Council of Ministers in the Centre: The Council of Ministers is an integral part of the parliamentary system of government in India. It consists of ministers who are appointed by the President of India on the advice of the Prime Minister. Let’s analyze each statement:

  1. The Council of Ministers in the Centre shall be collectively responsible to the Parliament: This statement is correct. The principle of collective responsibility means that all ministers in the Council are collectively responsible for the policies and decisions of the government. They are accountable to the Parliament and must collectively resign if they lose the confidence of the majority in the Lok Sabha (House of the People).
  2. The Union Ministers shall hold the office during the pleasure of the President of India: This statement is correct. The Union Ministers hold office at the pleasure of the President. This means they can be dismissed by the President on the advice of the Prime Minister at any time. However, in practice, the resignation of the Council of Ministers is tied to their majority support in the Lok Sabha.
  3. The Prime Minister shall communicate to the President about the proposals for legislation: This statement is correct. In the parliamentary system, the President acts on the advice of the Council of Ministers, with the Prime Minister as the head of the government. The Prime Minister advises the President on various matters, including the introduction of legislation in the Parliament.

69. Consider the following statements:

  1. National Development Council is an organ of the Planning Commission.
  2. The Economic and Social Planning is kept in the Concurrent List in the Constitution of India.
  3. The Constitution of India prescribes that Panchayats should be assigned the task of preparation of plans for economic development and social justice.

Which of the statements given above is/are correct?

(a) 1 only
(b) 2 and 3 only
(c) 1 and 3 only
(d) 1, 2 and 3

Answer: (a) 1 only

Explanation:

  1. National Development Council (NDC): The National Development Council was an advisory body that played a crucial role in the formulation of India’s Five-Year Plans. However, the statement that it is an organ of the Planning Commission is not correct. The National Development Council (NDC) was not an organ of the Planning Commission but a separate entity that included the Prime Minister, Chief Ministers of all states, and administrators of Union Territories. It was established to secure cooperation and coordination between the Central and State governments for the planned economic development of the country. The NDC was abolished in 2015.
  2. The Economic and Social Planning is kept in the Concurrent List in the Constitution of India: This statement is not correct. The Constitution of India has three lists: the Union List, the State List, and the Concurrent List. Economic and social planning, which falls under Entry 20 of the State List, is the exclusive domain of the state governments. The Central government plays a coordinating role through the Planning Commission (now replaced by the NITI Aayog).
  3. The Constitution of India prescribes that Panchayats should be assigned the task of preparation of plans for economic development and social justice: This statement is not correct. While the Constitution of India empowers Panchayats and Municipalities to prepare plans for economic development and social justice at the local level, the primary responsibility for national-level planning lies with the Central government and the Planning Commission (now replaced by the NITI Aayog).

70. Consider the following statements:

  1. The Chairman and the Deputy Chairman of the Rajya Sabha are not members of that House.
  2. While the nominated members of the two Houses of the Parliament have no voting right in the presidential election, they have the right to vote in the election of the Vice President.

Which of the statements given above is/are correct?

(a) 1 only
(b) 2 only
(c) Both 1 and 2
(d) Neither 1 nor 2

Answer: (b) 2 only

Explanation: The Chairman and the Deputy Chairman of the Rajya Sabha are not members of that House: This statement is not correct. The Chairman and Deputy Chairman of the Rajya Sabha are elected by the members of the Rajya Sabha from among its members. The Chairman is the presiding officer of the Rajya Sabha, and the Deputy Chairman acts as the Chairman in the absence of the Chairman.

  1. The Chairman and the Deputy Chairman of the Rajya Sabha are not members of that House: This statement is not correct. The Chairman and Deputy Chairman of the Rajya Sabha are elected by the members of the Rajya Sabha from among its members. The Chairman is the presiding officer of the Rajya Sabha, and the Deputy Chairman acts as the Chairman in the absence of the Chairman. Both the Chairman and Deputy Chairman are members of the Rajya Sabha.
  2. While the nominated members of the two Houses of the Parliament have no voting right in the presidential election, they have the right to vote in the election of the Vice President: This statement is correct. Nominated members of both the Rajya Sabha and the Lok Sabha do not have the right to participate in the election of the President of India. Only elected members of the Parliament (both Rajya Sabha and Lok Sabha) and the State Legislative Assemblies have voting rights in the presidential election.

71. With reference to the National Legal Services Authority, consider the following statements:

  1. Its objective is to provide free and competent legal services to the weaker sections of society on the basis of equal opportunity.
  2. It issues guidelines for the State Legal Services Authorities to implement legal programs and schemes throughout the country.

Which of the statements given above is/are correct?

(a) 1 only
(b) 2 only
(c) Both 1 and 2
(d) Neither 1 nor 2

Answer: (c) Both 1 and 2

Explanation:

National Legal Services Authority (NALSA): The National Legal Services Authority (NALSA) is a statutory body constituted under the Legal Services Authorities Act, 1987. Its primary objective is to provide free and competent legal services to the weaker sections of society, including marginalized and disadvantaged groups, women, children, Scheduled Castes, Scheduled Tribes, and other vulnerable populations.

  1. Its objective is to provide free and competent legal services to the weaker sections of society on the basis of equal opportunity: This statement is correct. NALSA’s primary purpose is to ensure access to justice for all, especially for those who are unable to afford legal services due to their economic or social circumstances. It aims to promote equality before the law and equal opportunity for justice to all citizens.
  2. It issues guidelines for the State Legal Services Authorities to implement legal programs and schemes throughout the country: This statement is correct. NALSA plays a crucial role in coordinating and monitoring the activities of State Legal Services Authorities (SLSAs) across the country. It issues guidelines, directions, and schemes to ensure uniformity in the implementation of legal services programs at the state level.

72. During a thunderstorm, the thunder in the skies is produced by the

  1. meeting of cumulonimbus clouds in the sky
  2. lightning that separates the nimbus clouds
  3. violent upward movement of air and water particles

Select the correct answer using the codes given below.

(a) 1 only
(b) 2 and 3
(c) 1 and 3
(d) None of the above produces the thunder

Answer: (d) None of the above produces the thunder

Explanation: Thunderstorms are severe weather phenomena associated with thunder, lightning, heavy rain, strong winds, and sometimes hail. Thunder is the sound produced by the rapid expansion of air surrounding a lightning bolt.

How thunder is produced during a thunderstorm:

  1. Formation of Lightning: Thunderstorms are characterized by the presence of cumulonimbus clouds, which are large and vertically developed clouds. Lightning is a discharge of electricity that occurs within these cumulonimbus clouds or between the cloud and the ground. During a thunderstorm, when conditions favor the buildup of electrical charges within the clouds, a lightning strike occurs.
  2. Lightning and Thunder: The rapid heating of the air surrounding a lightning bolt causes it to expand rapidly, creating a shockwave that produces the sound we hear as thunder. Thunder is essentially the sonic shockwave created by the explosive expansion of air due to the intense heat of the lightning discharge.
  3. Upward Movement of Air and Water Particles: The statement that the thunder is produced by the violent upward movement of air and water particles is not correct. 

73. Consider the following pairs:

Tribe – State

  1. Limboo (Limbu) : Sikkim
  2. Karbi : Himachal Pradesh
  3. Dongaria : Odisha
  4. Bonda : Tamil Nadu

Which of the above pairs are correctly matched?

(a) 1 and 3 only
(b) 2 and 4 only
(c) 1, 3, and 4 only
(d) 1, 2, 3, and 4

Answer: (a) 1 and 3 only

Explanation:

  1. Limboo (Limbu) : Sikkim: This pair is correctly matched. The Limboo (Limbu) tribe is a major tribe of Sikkim and is recognized as one of the indigenous communities of the state.
  2. Karbi : Himachal Pradesh: This pair is not correctly matched. The Karbi tribe is not from Himachal Pradesh. The Karbi people are an ethnic group living primarily in the Karbi Anglong district of Assam and parts of Nagaland.
  3. Dongaria : Odisha: This pair is correctly matched. The Dongaria Kondh tribe is a prominent tribal community of Odisha. They reside mainly in the Niyamgiri hills of Rayagada and Kalahandi districts.
  4. Bonda : Tamil Nadu: This pair is not correctly matched. The Bonda tribe is not from Tamil Nadu. The Bonda people, also known as the Bondo or Remo, are a tribal community living in the hills of the Malkangiri district of Odisha.

74. Consider the following liquid assets:

  1. Demand deposits with the banks
  2. Time deposits with the banks
  3. Savings deposits with the banks
  4. Currency

The correct sequence of these in decreasing order of Liquidity is

(a) 1-4-3-2
(b) 4-3-2-1
(c) 2-3-1-4
(d) 4-1-3-2

Answer: (a) 1-4-3-2

Explanation: Liquidity refers to the ease with which an asset can be converted into cash without significantly affecting its value. The liquidity of the given assets can be arranged in decreasing order as follows:

  1. Currency: Currency (cash) is the most liquid asset as it is physical money that can be used for transactions immediately without the need for any intermediary.
  2. Demand deposits with the banks: Demand deposits are accounts held in banks that allow depositors to withdraw their money on demand without any notice. They are highly liquid and can be easily converted into cash.
  3. Savings deposits with the banks: Savings deposits are accounts held in banks that pay interest to depositors but have some withdrawal restrictions. While they are still relatively liquid, there may be some restrictions on the number of withdrawals or minimum balance requirements.
  4. Time deposits with the banks: Time deposits, also known as fixed deposits, are accounts held in banks for a fixed term with a higher interest rate than savings accounts. However, they have a fixed maturity date, and withdrawing before maturity may result in penalties. Hence, they are less liquid compared to demand and savings deposits.

75. In the context of the Indian economy, “Open Market Operations” refers to

(a) borrowing by scheduled banks from the RBI
(b) lending by commercial banks to industry and trade
(c) purchase and sale of government securities by the RBI
(d) None of the above

Answer: (c) purchase and sale of government securities by the RBI

Explanation:

Open Market Operations (OMOs) are one of the monetary policy tools used by the Reserve Bank of India (RBI) to control the money supply and regulate liquidity in the economy. It involves the buying and selling of government securities in the open market.

  • Purchase of Government Securities: In an OMO purchase, the RBI buys government securities from banks or the public. By purchasing these securities, the RBI injects money into the banking system and increases the money supply. This action aims to increase liquidity and lower interest rates in the economy.
  • Sale of Government Securities: In an OMO sale, the RBI sells government securities to banks or the public. By selling these securities, the RBI absorbs excess money from the banking system and reduces the money supply. This action aims to reduce liquidity and increase interest rates in the economy.
  • Objectives of OMOs: The main objectives of conducting OMOs are to regulate short-term interest rates, manage liquidity in the banking system, and influence credit conditions in the economy to achieve the desired level of economic growth and stability.

76. Priority Sector Lending by banks in India constitutes the lending to

(a) agriculture
(b) micro and small enterprises
(c) weaker sections
(d) All of the above

Answer: (d) All of the above

Explanation: Priority Sector Lending (PSL) refers to the lending targets set by the Reserve Bank of India (RBI) for banks to ensure that a certain portion of their loans and advances is directed to specified sectors that are considered important for the overall development of the economy. The objective of PSL is to ensure equitable and inclusive growth by providing credit to vulnerable sectors and segments of the population.

The priority sectors identified for PSL include:

  1. Agriculture: Loans to agriculture and allied activities, including crop loans, term loans for agriculture-related activities, and agricultural infrastructure.
  2. Micro and Small Enterprises (MSEs): Loans to micro and small enterprises, including micro and small manufacturing units, service enterprises, and other small businesses.
  3. Weaker Sections: Loans to specific disadvantaged sections of society, such as Scheduled Castes (SCs), Scheduled Tribes (STs), Other Backward Classes (OBCs), minorities, and low-income groups.
  4. Other Priority Sectors: Besides the above sectors, PSL also includes lending to sectors like education, housing, renewable energy, export credit, and social infrastructure.
  • Conclusion: Therefore, Priority Sector Lending by banks in India constitutes the lending to all of the above sectors, i.e., (d) All of the above.

77. Which one among the following industries is the maximum consumer of water in India?

(a) Engineering
(b) Paper and pulp
(c) Textiles
(d) Thermal power

Answer: (d) Thermal power

Explanation: Among the given options, thermal power is the maximum consumer of water in India. The production of electricity from thermal power plants requires significant amounts of water for cooling purposes.

Water Usage in Thermal Power Plants:

  1. Cooling of Power Plants: Most thermal power plants use water as a coolant in their steam turbine-based electricity generation process. Water is required for cooling purposes to condense the steam back into water after it has passed through the turbine. This process is crucial for the efficient functioning of the power plant.
  2. Once-through Cooling vs. Cooling Towers: Some power plants adopt a once-through cooling system, where water is drawn from a nearby water body (river or sea) and discharged back after use. Others use cooling towers, which recirculate the water, reducing the overall water consumption but increasing evaporation.
  3. Water Consumption: The amount of water consumed by thermal power plants depends on the type of cooling system used, the plant’s capacity, and the local water availability.

Environmental Concerns:

  • The large water consumption by thermal power plants has raised concerns about water scarcity and environmental impacts. In areas facing water stress, the excessive use of water by power plants can strain local water resources.
  • Additionally, thermal power plants that use once-through cooling systems may discharge heated water back into water bodies, leading to thermal pollution, which can affect aquatic ecosystems.
  • Conclusion: Therefore, among the given industries, thermal power is the maximum consumer of water in India (option d).

78. To obtain full benefits of demographic dividend, what should India do?

(a) Promoting skill development
(b) Introducing more social security schemes
(c) Reducing infant mortality rate
(d) Privatization of higher education

Answer: (a) Promoting skill development

Explanation: 

Demographic Dividend: Demographic dividend refers to the period when a country experiences a significant proportion of its population in the working-age group (15-64 years), and the dependency ratio (proportion of the non-working-age population to the working-age population) is low. During this phase, there is a potential for economic growth and development due to a large and productive workforce.

To obtain full benefits of demographic dividend, India should focus on:

Promoting Skill Development: Enhancing the skills and employability of the working-age population is crucial for leveraging the demographic dividend. India needs to invest in skill development programs, vocational training, and education that align with the requirements of the job market. This will enable the workforce to be more productive, adaptable to changing technologies, and better equipped to contribute to economic growth.

Other Relevant Measures:

While promoting skill development is essential, other measures can also contribute to realizing the full benefits of demographic dividend:

  • Investing in Education: Improving the quality and access to education at all levels is essential. A well-educated population is better equipped to take advantage of economic opportunities and contribute to innovation and productivity.
  • Creating Employment Opportunities: Generating sufficient employment opportunities is vital to absorb the increasing number of young people entering the workforce. Encouraging entrepreneurship, supporting small and medium-sized enterprises, and promoting labor-intensive industries can help in this regard.
  • Infrastructure Development: Adequate infrastructure, including transportation, communication, and power, is necessary to support economic activities and attract investments.
  • Healthcare and Family Planning: Ensuring good healthcare facilities and family planning services can contribute to lowering the fertility rate, leading to a more favorable dependency ratio.
  • Social Security and Safety Nets: While not directly related to demographic dividend, enhancing social security schemes and safety nets can provide a sense of security to the workforce and promote overall well-being.
  • Conclusion: In summary, to obtain full benefits of demographic dividend, India should focus on promoting skill development, along with other complementary measures, to harness the potential of its young and productive workforce for economic growth and development.

79. In the context of cultural history of India, a pose in dance and dramatics called ‘Tribhanga’ has been a favorite of Indian artists from ancient times till today. Which one of the following statements best describes this pose?

(a) One leg is bent and the body is slightly but oppositely curved at the waist and neck
(b) Facial expressions, hand gestures, and make-up are combined to symbolize certain epic or historic characters
(c) Movements of body, face, and hands are used to express oneself or to tell a story
(d) A little smile, slightly curved waist, and certain hand gestures are emphasized to express the feelings of love or eroticism

Answer: (a) One leg is bent and the body is slightly but oppositely curved at the waist and neck

Explanation: Tribhanga is a pose in Indian classical dance and dramatics that has been a favorite of Indian artists from ancient times till today. The term “Tribhanga” is derived from Sanskrit, where “Tri” means three, and “Bhanga” means a bend or curve. The pose involves a three-fold or triple bend in the body, creating an S-shaped or serpentine form.

Characteristics of Tribhanga Pose:

In the Tribhanga pose:

  1. One Leg is Bent: The body weight is primarily supported by one leg, and the other leg is bent, giving the pose a dynamic and asymmetrical appearance.
  2. Oppositely Curved at the Waist and Neck: The waist and neck are gracefully curved in opposite directions, creating a gentle S-shaped curve in the body.
  3. Upper Body and Head Tilt: The upper body is inclined in a slightly diagonal manner, and the head is tilted in the opposite direction, adding to the elegant and expressive nature of the pose.

Importance in Dance and Dramatics:

The Tribhanga pose is considered one of the most graceful and expressive poses in Indian classical dance forms such as Odissi, Manipuri, and Bharatanatyam. It is often used to depict the divine or celestial beings, such as the gods and goddesses in Indian mythology. The pose is also frequently used to portray the emotional and narrative aspects of dance, conveying various moods and stories through body language and facial expressions.

80. Annie Besant was

  1. responsible for starting the Home Rule Movement
  2. the founder of the Theosophical Society
  3. once the President of the Indian National Congress

Select the correct statement/statements using the codes given below.

(a) 1 only (b) 2 and 3 only (c) 1 and 3 only (d) 1, 2, and 3

Answer: (b) 2 and 3 only

Explanation:

Annie Besant was a prominent British socialist, women’s rights activist, and supporter of Indian self-rule. The correct statements regarding her are as follows:

  1. The Founder of the Theosophical Society: Annie Besant was indeed the co-founder of the Theosophical Society along with Henry Steel Olcott in 1875. The Theosophical Society is a spiritual and philosophical organization that seeks to promote universal brotherhood and understanding between all people, irrespective of nationality, race, or religion.
  2. Once the President of the Indian National Congress: Annie Besant was actively involved in the Indian freedom struggle and served as the President of the Indian National Congress in 1917. She was the first woman to hold this position in the history of the Indian National Congress.
  3. Responsible for Starting the Home Rule Movement: This statement is incorrect. The Home Rule Movement was started by two prominent Indian leaders, Bal Gangadhar Tilak and Annie Besant herself. In September 1916, both of them jointly launched the Home Rule League in India, with Tilak leading the movement in the Maharashtra region and Besant leading it in other parts of India. The movement aimed to demand self-government or home rule for India within the British Empire.

21. The Parliament can make any law for whole or any part of India for implementing international treaties

(a) with the consent of all the States
(b) with the consent of the majority of States
(c) with the consent of the States concerned
(d) without the consent of any State

22. In the grasslands, trees do not replace the grasses as a part of an ecological succession because of

(a) insects and fungi
(b) limited sunlight and paucity of nutrients
(c) water limits and fire
(d) None of the above

23. Which one of the following is the correct sequence of ecosystems in the order of decreasing productivity?

(a) Oceans, lakes, grasslands, mangroves
(b) Mangroves, oceans, grasslands, lakes
(c) Mangroves, grasslands, lakes, oceans
(d) Oceans, mangroves, lakes, grasslands

24. Contour bunding is a method of soil conservation used in

(a) desert margins, liable to strong wind action
(b) low flat plains, close to stream courses, liable to flooding
(c) scrublands, liable to spread of weed growth
(d) None of the above

25. The Government enacted the Panchayat Extension to Scheduled Areas (PESA) Act in 1996. Which one of the following is not identified as its objective?

(a) To provide self-governance
(b) To recognize traditional rights
(c) To create autonomous regions in tribal areas
(d) To free tribal people from exploitation

26. Under the Scheduled Tribes and Other Traditional Forest Dwellers (Recognition of Forest Rights) Act, 2006, who shall be the authority to initiate the process for determining the nature and extent of individual or community forest rights or both?

(a) State Forest Department
(b) District Collector/Deputy Commissioner
(c) Tahsildar /Block Development Officer / Mandai Revenue Officer
(d) Gram Sabha

27. Improper handling and storage of cereal grains and oilseeds result in the production of toxins known as aflatoxins which are not generally destroyed by normal cooking process. Aflatoxins are produced by

(a) bacteria
(b) protozoa
(c) moulds
(d) viruses

28. ‘Economic Justice’ the objectives of Constitution has been as one of the Indian provided in

(a) the Preamble and Fundamental Rights
(b) the Preamble and the Directive Principles of State Policy
(c) the Fundamental Rights and the Directive Principles of State Policy
(d) None of the above

29. Due to improper / indiscriminate disposal of old and used computers or their parts, which of the following are released into the environment as e-waste?

1. Beryllium
2. Cadmium
3. Chromium
4. Heptachlor
5. Mercury
6. Lead
7. Plutonium

Select the correct answer using the codes given below.

(a) 1, 3, 4, 6 and 7 only
(b) 1, 2, 3, 5 and 6 only
(c) 2, 4, 5 and 7 only
(d) 1, 2, 3, 4, 5, 6 and 7

30. Acid rain is caused by the pollution of environment by

(a) carbon dioxide and nitrogen
(b) carbon monoxide and carbon dioxide
(c) ozone and carbon dioxide
(d) nitrous oxide and sulphur dioxide

31. With reference to food chains in ecosystems, consider the following statements :

1. A food chain illustrates the order in which a chain of organisms feed upon each other.
2. Food chains are found within the populations of a species.
3. A food chain illustrates the numbers of each organism which are eaten by others.

Which of the statements given above is / are correct?

(a) 1 only
(b) 1 and 2 only
(c) 1, 2 and 3
(d) None

32. Consider the following pairs

National Park   –   River flowing through the Park
1. Corbett National Park   :   Ganga
2. Kaziranga National Park   :   Manas
3. Silent Valley National Park   :   Kaveri

Which of the above pairs is/are correctly matched?

(a) 1 and 2
(b) 3 only
(c) 1 and 3
(d) None

33. Consider the following organisms

1. Agaricus
2. Nostoc
3. Spirogyra

Which of the above is / are used as bio fertilizer / bio fertilizers

(a) 1 and 2
(b) 2 only
(c) 2 and 3
(d) 3 only

34. Which of the following adds / add nitrogen to the soil?

1. Excretion of urea by animals
2. Burning of coal by man
3. Death of vegetation

Select the correct answer using the codes given below.

(a) 1 only
(b) 2 and 3 only
(e) 1 and 3 only
(d) 1, 2 and 3

35. In which of the following States is lion-tailed macaque found in its natural habitat?

1. Tamil Nadu
2. Kerala
3. Karnataka
4. Andhra Pradesh

Select the correct answer using the codes given below.

(a) 1, 2 and 3 only
(b) 2 only
(c) 1, 3 and 4 only
(d) 1, 2, 3 and 4

36. Some Buddhist rock-cut caves are called Chaityas, while the others are called Viharas. What is the difference between the two?

(a) Vihara is a place of worship, while Chaitya is the dwelling place of the monks
(b) Chaitya is a place of worship, while Vihara is the dwelling place of the monks
(c) Chaitya is the stupa at the far end of the cave, while Vihara is the hall axial to it
(d) There is no material difference between the two

37. Which one of the following describes best the concept of Nirvana in Buddhism?

(a) The extinction of the flame of desire
(b) The complete annihilation of self
(c) A state of bliss and rest
(d) A mental stage beyond all comprehension

38. According to the Constitution of India, which of the following are fundamental for the governance of the country?

(a) Fundamental Rights
(b) Fundamental Duties
(c) Directive Principles of State Policy
(d) Fundamental Rights and Fundamental Duties

39. The people of India agitated against the arrival of Simon Commission because

(a) Indians never wanted the review of the working of the Act of 1919
(b) Simon Commission recommended the abolition of Dyarchy (Diarchy) in the Provinces
(c) there was no Indian member in the Simon Commission
(d) the Simon Commission suggested the partition of the country

40. Quit India Movement was launched in response to

(a) Cabinet Mission Plan
(b) Cripps Proposals
(c) Simon Commission Report
(d) Wavell Plan

41. The balance of payments of a country is a systematic record of

(a) all import and transactions of a during a given period normally a year
(b) goods exported from a country during a year
(c) economic transaction between the government of one country to another
(d) capital movements from one country to another

42. The Reserve Bank of India regulates the commercial banks in matters of

1. liquidity of assets
2. branch expansion
3. merger of banks
4. winding-up of banks

Select the correct answer using the codes given below.

(a) 1 and 4 only
(b) 2, 3 and 4 only
(c) 1, 2 and 3 only
(d) 1, 2, 3 and 4

43. An increase in the Bank Rate generally indicates that the

(a) market rate of interest is likely to fall
(b) Central Bank is no longer making loans to commercial banks
(c) Central Bank is following an easy money policy
(d) Central Bank is following a tight money policy

44. In India, deficit financing is used for raising resources for

(a) economic development
(b) redemption of public debt
(c) adjusting the balance of payments
(d) reducing the foreign debt

45. Which of the following characterizes/ characterize the people of Indus Civilization?

1. They possessed great palaces and temples.
2. They worshipped both male and female deities.
3. They employed horse-drawn chariots in warfare.

Select the correct statement/ statements using the codes given below.

(a) 1 and 2 only
(b) 2 only
(e) 1, 2 and 3
(d) None of the statements given above is correct

46. Which of the following diseases can be transmitted from one person to another through tattooing?

1. Chikungunya
2. Hepatitis B
3. HIV-AIDS

Select the correct answer using the codes given below.

(a) 1 and 2 only
(b) 2 only
(e) 1, 2 and 3
(d) None of the statements given above is correct

47. Which of the following statements is/are applicable to Jain doctrine?

1. The surest way of annihilating Karma is to practice penance.
2. Every object, even the smallest particle has a soul.
3. Karma is the bane of the soul and must be ended.

Select the correct answer using the codes given below.

(a) 1 only
(b) 2 and 3 only
(c) 1 and 3 only
(d) 1, 2 and 3

48. Which one of the following terms describes not only the physical space occupied by an organism, but also its functional role in the community of organisms?

(a) Ecotone
(b) Ecological niche
(c) Habitat
(d) Home range

49. Photochemical smog is a resultant of the reaction among

(a) NO2, 03 and peroxyacetyl nitrate in the presence of sunlight
(b) CO, 02 and peroxyacetyl nitrate in the presence of sunlight
(c) CO, CO2 and N02 at low temperature
(d) High concentration of N02, O3 and CO in the evening

50. Consider the following minerals

1. Calcium
2. Iron
3. Sodium

Which of the minerals given above is/are required by human body for the contraction of muscles?

(a) 1 only
(b) 2 and 3 only
(c) 1 and 3 only
(d) 1, 2 and 3

51. Consider the following statements: The Parliamentary Committee on Public Accounts

1. consists of not more than 25 Members of the Lok Sabha
2. scrutinizes appropriation finance accounts of Government
3. of the Auditor examines the report Comptroller and General of India

Which of the statements given above is / are correct?

(a) 1 only
(b) 2 and 3 only
(e) 3 only
(d) 1, 2 and 3

52. Consider the following Bhakti Saints:

1. Dadu Dayal
2. Guru Nanak
3. Tyagaraja

Who among the above was/were preaching when the Lodi dynasty fell and Babur took over?

(a) 1 and 3
(b) 2 only
(c) 2 and 3
(d) 1 and 2

53. With reference to the food chains in ecosystems, which of the following kinds of organism is / are known as decomposer organism/organisms?

1. Virus
2. Fungi
3. Bacteria

Select the correct answer using the codes given below.

(a) 1 only
(b) 2 and 3 only
(c) 1 and 3 only
(d) 1, 2 and 3

54. The most important fishing grounds of the world are found in the regions where

(a) warm and cold atmospheric currents meet
(b) rivers drain out large amounts of fresh water into the sea
(c) warm and cold oceanic currents meet
(d) continental shelf is undulating

55. Which of the following is/are unique characteristic/characteristics of equatorial forests?

1. Presence of tall, closely set trees with crowns forming a continuous canopy
2. Coexistence of a large number of species
3. Presence of numerous varieties of epiphytes

Select the correct answer using the code given below:

(a) 1 only
(b) 2 and 3 only
(c) 1 and 3 only
(d) 1, 2 and 3

56. Which of the following constitute Capital Account?

1. Foreign Loans
2. Foreign Direct Investment
3. Private Remittances
4. Portfolio Investment

Select the correct answer using the codes given below.

(a) 1, 2 and 3
(b) 1, 2 and 4
(c) 2, 3 and 4
(d) 1, 3 and 4

57. Consider the following historical places:

1. Ajanta Caves
2. Lepakshi Temple
3. Sanchi Stupa

Which of the above places is / are also known for mural paintings?

(a) 1 only
(b) 1 and 2 only
(c) 1, 2 and 3
(d) None

58. With reference to the history of philosophical thought in India, consider the following statements regarding Sankhya school:

1. Sankhya does not accept the theory of rebirth or transrmigration of soul.
2. Sankhya holds that it is the self-knowledge that leads to liberation and not any exterior influence or agent.

Which of the statements given above is /are correct?

(a) 1 only
(b) 2 only
(c) Both 1 and 2
(d) Neither 1 nor 2

59. In the context of India, which of the following principles is/are implied institutionally in the parliamentary government?

1. Members of the Cabinet are Members of the Parliament.
2. Ministers hold the office till they enjoy confidence in the Parliament.
3. Cabinet is headed by the Head of the State.

Select the correct answer using the codes given below.

(a) 1 and 2 only
(b) 3 only
(c) 2 and 3 only
(d) 1, 2 and 3

60. The annual range of temperature in the interior of the continents is high as compared to coastal areas. What is / are the reason / reasons?

1. Thermal difference between land and water
2. Variation in altitude between continents and oceans
3. Presence of strong winds in the interior
4. Heavy rains in the interior as compared to coasts

Select the correct answer using the codes given below.

(a) 1 only
(b) 1 and 2 only
(c) 2 and 3 only
(d) 1, 2, 3 and 4

61. Which of the following is / are the characteristic/ characteristics of Indian coal?

1. High ash content
2. Low sulphur content
3. Low ash fusion temperature

Select the correct answer using the codes given below.

(a) 1 and 2 only
(b) 2 only
(c) 1 and 3 only
(d) 1, 2 and 3

62. Which of the following statements regarding laterite soils of India are correct?

1. They are generally red in colour.
2. They are rich In nitrogen and potash.
3. Tiley are well-developed in Rajasthan and UP.
4. Tapioca and cashew nuts grow well on these soils.

Select the correct answer using the codes given below.

(a) 1, 2 and 3
(b) 2, 3 and 4
(c) 1 and 4
(d) 2 and 3 only

63. Consider the following statements:

1. Natural gas occurs in the Gondwana beds.
2. Mica occurs in abundance in Kodarma.
3. Dharwars are famous for petroleum.

Which of the statements given above is/are correct?

(a) 1 and 2
(b) 2 only
(c) 2 and 3
(d) None

64. Consider the following crops

1. Cotton
2. Groundnut
3. Rice
4. Wheat

Which of these are Kharif crops?

(a) 1 and 4
(b) 2 and 3 only
(c) 1, 2 and 3
(d) 2, 3 and 4

65. “Climate is extreme, rainfall is scanty and the people used to be nomadic herders.” The above statement best describes which of the following regions?

(a) African Savannah
(b) Central Asian Steppe
(c) North American Prairie
(d) Siberian Tundra

66. Consider the following statements :

1. Inflation benefits the debtors.
2. Inflation benefits the bond-holders.

Which of the statements given above is/are correct?

(a) 1 only
(b) 2 only
(c) Both 1 and 2
(d) Neither 1 nor 2

67. Disguised unemployment generally means

(a) large number of people remain unemployed
(b) alternative employment is not available
(c) marginal productivity of labour is zero
(d) productivity of workers is low

68. Consider the following statements:

1. The Council of Ministers in the Centre shall be collectively responsible to the Parliament.
2. The Union Ministers shall hold the office during the pleasure of the President of India.
3. The Prime Minister shall communicate to the President about the proposals for Iegislation.

Which of the Statements given above is/are correct?

(a) 1 only
(b) 2 and 3 only
(c) 1 and 3 only
(d) 1,2 and 3

69. Consider the following statements:

1. National Development Council is an organ of the Planning Commission.
2. The Economic and Social Planning is kept in the Concurrent List in the Constitution of India.
3. The Constitution of India prescribes that Panchayats should be assigned the task of preparation of plans for economic development and social justice.

Which of the statements given above is/are correct?

(a) 1 only
(b) 2 and 3 only
(c) 1 and 3 only
(d) 1, 2 and 3

70. Consider the following statements:

1. The Chairman and the Deputy Chairman of the Rajya Sabha are not the members of that House.
2. While the nominated members of the two Houses of the Parliament have no voting right in the presidential election, they have the right to vote in the election of the Vice President.

Which of the statements given above is/are correct?

(a) 1 only
(b) 2 only
(c) Both 1 and 2
(d) Neither 1 nor 2

71. With reference to National Legal Services Authority, consider the following statements :

1. Its objective is to provide free and competent legal services to the weaker sections of the society on the basis of equal opportunity.
2. It issues guidelines for the State Legal Services Authorities to implement the legal programmes and schemes throughout the country.

Which of the statements given above is / are correct?

(a) 1 only
(b) 2 only
(C) Both 1 and 2
(d) Neither 1 nor 2

72. During a thunderstorm, the thunder in the skies is produced by the

1. meeting of cumulonimbus clouds in the sky
2. lightning that separates the nimbus clouds
3. violent upward movement of air and water particles

Select the correct answer using the codes given below.

(a) 1 only
(b) 2 and 3
(C) 1 and 3
(d) None of the above produces the thunder

73. Consider the following pairs :

Tribe   –   State
1. Limboo (Limbu)   :   Sikkim
2. Karbi   :   Himachal Pradesh
3. Dongaria   :   Odisha
4. Bonda   :   Tamil Nadu

Which of the above pairs are correctly matched?

(a) 1 and 3 only
(b) 2 and 4 only
(c) 1, 3 and 4 only
(d) 1, 2, 3 and 4

74. Consider the following liquid assets:

1. Demand deposits with the banks
2. Time deposits with the banks
3. Savings deposits with the banks
4. Currency

The correct sequence of these decreasing order of Liquidity is

(a) 1-4-3-2
(b) 4-3-2-1
(c) 2-3-1-4
(d) 4-1-3-2

75. In the context of Indian economy, Open Market Operations’ refers to

(a) borrowing by scheduled banks from the RBI
(b) lending by commercial banks to industry and trade
(c) purchase and sale of government securities by the RBI
(d) None of the above

76. Priority Sector Lending by banks in India constitutes the lending to

(a) agriculture
(b) micro and small enterprises
(c) weaker sections
(d) All of the above

77. Which one among the following industries is the maximum consumer of water in India?

(a) Engineering
(b) Paper and pulp
(e) Textiles
(d) Thermal power

78. To obtain full benefits of demographic dividend, what should India do?

(a) Promoting skill development
(b) Introducing more social security schemes
(c) Reducing infant mortality rate
(d) Privatization of higher education

79. In the context of cultural history of India, a pose in dance and dramatics called ‘Tribhanga’ has been a favourite of Indian artists from ancient times till today. Which one of the following statements best describes this pose?

(a) One leg is bent and the body is slightly but oppositely curved at waist and neck
(b) Facial expressions, hand gestures and make-up are combined to symbolize certain epic or historic characters
(c) Movements of body, face and hands are used to express oneself or to tell a story
(d) A little smile, slightly curved waist and certain hand gestures are emphasized to express the feelings of love or eroticism

80. Annie Besant was

1. responsible for starting the Home Rule Movement
2. the founder of the Theosophical Society
3. once the President of the Indian National Congress

Select the correct statement/statements using the codes given below.

(a) 1 only
(b) 2 and 3 only
(c) 1 and 3 only
(d) 1, 2 and 3

81. The Ilbert Bill controversy was related to the

(a) imposition restrictions the Indians of certain to carry arms by Indians
(b) imposition of restrictions on newspapers and magazines published in Indian languages
(c) removal of disqualifications imposed on the Indian magistrates with regard to the trial of the Europeans
(d) removal of a duty on imported cotton cloth

82. A rise in general level of prices may be caused by

1. an increase in the money supply
2. a decrease in the aggregate level of output
3. an increase in the effective demand

Select the correct answer using the codes given below.

(a) 1 only
(b) 1 and 2 only
(c) 2 and 3 only
(d) 1, 2 and 3

83. Which one of the following groups of items is included in India’s foreign-exchange reserves?

(a) Foreign-currency assets, Special Drawing Rights (SDRs) and loans from foreign countries
(b) Foreign-currency assets, gold oldings of the RBI and SDRs
(c) Foreign-currency assets, loans from the World Bank and SDRs
(d) Foreign-currency assets, gold holdings of the RBI and loans from the World Bank

84. Which one of the following is likely to be the most inflationary in its effect?

(a) Repayment of public debt
(b) Borrowing from the public to finance a budget deficit
(c) Borrowing from banks to finance a budget deficit
(d) Creating new money to finance a budget deficit

85. Supply of money remammg the same when there is an increase in demand for money, there will be

(a) a fall in the level of prices
(b) an increase in the rate of interest
(c) a decrease in the rate of interest
(d) an increase in the level of income and employment

86. Fruits stored in a cold chamber exhibit longer storage life because

(a) exposure to sunlight is prevented
(b) concentration dioxide in the is increased of carbon environment
(c) rate of respiration is decreased
(d) there is an increase in humidity

87. Consider the following fauna of India:

1. Gharial
2. Leatherback turtle
3. Swamp deer

Which of the above is/are endangered?

(a) 1 and 2 only
(b) 3 only
(c) 1, 2 and 3
(d) None

88. Ball bearings are used in bicycles, cars, etc., because

(a) the actual between the is increased area of contact wheel and axle
(b) the effective between the is increased area of contact wheel and axle
(c) the effective between the is reduced area of contact wheel and axle
(d) None of the above statements is correct

89. Consider the following phenomena:

1. Size of the sun at dusk
2. Colour of the sun at dawn
3. Moon being visible at dawn
4. Twinkle of stars in the sky
5. Polestar beingvisible in the sky

Which of the above are optical illusions?

(a) 1, 2 and 3
(b) 3, 4 and 5
(c) 1, 2 and 4
(d) 2, 3 and 5

90. Rainbow is produced when sunlight falls on drops of rain. Which of the following physical phenomena are responsible for this?

1. Dispersion
2. Refraction
3. Internal reflection

Select the correct answer using the codes given below.

(a) 1 and 2 only
(b) 2 and 3 only
(c) 1 and 3 only
(d) 1, 2 and 3

91. Many transplanted seedlings do not grow because

(a) the new soil does not contain favourable minerals
(b) most of the root hairs grip the new soil too hard
(c) most of the root hairs are lost during transplantation
(d) leaves get damaged during transplantation

92. Economic growth in country X will necessarily have to occur if

(a) there is technical progress in the world economy
(b) there is population growth in X
(c) there is capital formation in X
(d) the volume of trade grows in the world economy

93. Which of the following statements is / are correct?

1. Viruses lack enzymes necessary for the generation of energy.
2. Viruses can be cultured in any synthetic medium.
3. Viruses are transmitted from one organism to another by biological vectors only.

Select the correct answer using the codes given below.

(a) 1 only
(b) 2 and 3 only
(c) 1 and 3 only
(d) 1, 2 and 3

Ans: (a)

Solution: S1: Viruses are obligate intracellular parasites that cannot produce energy on their own. They do not have the metabolic machinery to generate ATP (adenosine triphosphate), which is the energy currency of the cell. Therefore, they rely on the host cell’s metabolic machinery to produce energy and synthesize new viral particles.

S2: Viruses require a living host cell to replicate, and therefore, they cannot be cultured in synthetic media. However, some viruses can be grown in tissue culture, which involves culturing host cells in vitro and infecting them with the virus. This technique allows researchers to study virus-host interactions and develop vaccines.

S3: Viruses can be transmitted through both biological and non-biological vectors. Biological vectors are living organisms such as mosquitoes, ticks, and rodents, which transmit the virus from one host to another. Non-biological vectors include contaminated food, water, and fomites (inanimate objects), which can also spread the virus. 

94. Which of the following leaf modifications occurs/occur in desert areas to inhibit water loss?

1. Hard and waxy leaves
2. Tiny leaves or no leaves
3. Thorns instead of leaves

Select the correct answer using the codes given below.

(a) 1 and 2 only
(b) 2 only
(c) 1 and 3 only
(d) 1, 2 and 3

Ans: (d)

Solution: In desert areas, plants have evolved various adaptations to conserve water. Some of these adaptations include:

  1. Hard and waxy leaves: This helps to reduce water loss through transpiration by creating a barrier to prevent water from escaping the plant.
  2. Tiny leaves or no leaves: This helps to reduce the surface area through which water can be lost via transpiration.
  3. Thorns instead of leaves: In some desert plants, thorns have replaced leaves to reduce the surface area through which water can be lost, and to protect the plant from herbivores.

95. The known forces of nature can be divided into four classes, viz, gravity electromagnetism, weak nuclear force and strong nuclear force. with reference to them, which one of the following statements is not correct?

(a) Gravity is the strongest of the four
(b) Electromagnetism act only on particles with an electric charge
(c) Weak nuclear force causes radioactivity
(d) Strong nuclear force holds protons and neutrons inside the nuclear of an atom.

Ans: (a)

Solution: Gravity is the weakest of the four fundamental forces. It is the force of attraction that exists between any two masses, any two bodies, any two particles. Electromagnetic force, on the other hand, acts on all particles with an electric charge, whether they are moving or not. The strong nuclear force holds the nucleus of an atom together by acting between the protons and neutrons, while the weak nuclear force is responsible for nuclear decay and radioactivity.

96. The efforts to detect the existence of Higgs boson particle have become frequent news in the recent past. What is /are the importance/importances of discovering this particle?

1. It will enable us to understand as to why elementary particles have mass.
2. It will enables us in the near future to develop the technology to transferring matter from one point to another without traversing the physical space between them.
3. It will enable us to create better fuels for nuclear fission.

Select the correct answer using the codes given below:

(a) 1 only
(b) 2 and 3 only
(c) 1 and 3 only
(d) 1, 2 and 3

Ans: (a)

Solution: The discovery of the Higgs boson particle is important because it confirms the existence of the Higgs field, which is responsible for giving elementary particles mass. This discovery is a crucial step towards a more complete understanding of the fundamental nature of matter and the universe. It is not directly related to the development of technology for matter transfer or creating better fuels for nuclear fission.

97. Mycorrhizal biotechnology has been used in rehabilitating degraded sites because mycorrhiza enables the plants to

1. resist drought and increase absorptive area
2. tolerate extremes of PH
3. Resist disease infestation

Select the correct answer using the codes given below:

(a) 1 only
(b) 2 and 3 only
(c) 1 and 3 only
(d) 1, 2 and 3

Ans: (D)

98. Who among the following constitute the National Development Council?

1. The Prime Minister
2. The Chairman, Finance Commission
3. Ministers of the Union Cabinet
4. Chief Ministers of the States

Select the correct answer using the codes given below:

(a) 1, 2 and 3 only
(b) 1, 3 and 4 only
(c) 2 and 4 only
(d) 1, 2, 3 and 4

Ans: (b)

Solution: The National Development Council (NDC) is an apex body constituted in India to strengthen and foster the cooperative federalism model of governance. The NDC is chaired by the Prime Minister of India and includes the following members:

  1. The Prime Minister
  2. Union Cabinet Ministers
  3. Chief Ministers of all the states
  4. Administrators of Union Territories.

The Chairman of the Finance Commission of India is also invited to participate in the meetings of the NDC as a special invitee.  However, he is not a part of it.

99. The national income of a country for a given period is equal to the

(a) total value of goods and services produced by the nationals
(b) sum of total consumption and investment expenditure
(c) sum of personal income of all individuals
(d) money value of final goods and services produced

Ans: (d)

Solution: The national income of a country represents the total value of goods and services produced within the country’s borders during a given period. It includes the money value of all final goods and services produced by all sectors of the economy, such as agriculture, industry, and services. It is usually measured using Gross Domestic Product (GDP) or Gross National Product (GNP).

100. Which of the following grants/ grant direct credit assistance to rural households ?

1. Regional Rural Banks
2. National Bank for Agriculture and Rural Development
3. Land Development Banks

Select the correct anser using the codes given below:

(a) 1 and 2 only
(b) 2 only
(c) 1 and 3 only
(d) 1, 2 and 3

Ans: (c)

Solution: 

S1: Regional Rural Banks (RRBs) were established in India in 1975 to provide credit and other financial services to rural areas. They were established under the provisions of the Regional Rural Banks Act of 1976 and are jointly owned by the Central Government, the concerned State Government and the sponsoring bank. The main objective of RRBs is to provide credit and other financial facilities to the small and marginal farmers, agricultural labourers, artisans, and other rural people who lack access to formal credit channels. RRBs are supervised by the National Bank for Agriculture and Rural Development (NABARD). Hence, Option 1 gets included. 

S2: The National Bank for Agriculture and Rural Development (NABARD) is a development financial institution in India that was established in 1982. It is headquartered in Mumbai and has regional offices throughout the country. NABARD’s main objective is to promote sustainable and equitable agriculture and rural development in India by providing credit and other financial services to rural areas.

NABARD provides credit to various rural development agencies, cooperatives, and regional rural banks, and also refinances loans given by commercial banks and other financial institutions for agricultural and rural development purposes.

Hence, S2 gets rejected. 

S3: Land Development Banks (LDBs) are financial institutions that specialize in providing long-term credit to farmers for agricultural development. They were established in India during the 1960s to promote agricultural credit and rural development.

LDBs operate at the state level and are typically owned by the state government, with the participation of the Central government and farmers’ cooperatives. They provide loans to farmers for various agricultural purposes such as land development, purchase of agricultural equipment, and other related activities. The loans are typically given at low-interest rates and with flexible repayment options to enable farmers to repay the loans easily.

The primary objective of LDBs is to provide credit to farmers for agricultural development and help in improving their socio-economic condition.

So, S3 is included. 

 

Leave a Comment

Your email address will not be published. Required fields are marked *